BCBA exam questions

Lakukan tugas rumah & ujian kamu dengan baik sekarang menggunakan Quizwiz!

Daphne's client has asked her to share behavior data with their neurological pediatrician to help inform possible medication for their daughter. What does Daphne need in order to do so? A)A completed request form from the pediatrician.B)HIPAA regulations prevent the sharing of data of any kind.C)A written consent from the client or client's guardian for disclosure.D)A signed waiver authorizing the BCBA® to share documents with their pediatrician.

C. Behavior analysts obtain the written consent of the client before obtaining or disclosing client records from or to other sources, for assessment purposes.

This level of scientific study is achieved when observations reveal that two events consistently covary with each other: A)DescriptionB)CorrelationC)PredictionD)Control

C. Prediction occurs when repeated observations reveal that two events consistently covary with each other. In other words, prediction helps us establish relations between events. "Description" and "Control" are two other levels of scientific study, but do not apply. "Correlation" is a systematic covariation between two events.

Which of the following statements is/are TRUE regarding time sampling recording procedures? A)Time sampling recording procedures are also known as discontinuous recording procedures.B)Time sampling recording procedures include partial-interval recording, whole-interval recording, and momentary time sampling.C)Time sampling recording procedures are particularly useful when measuring behaviors that do not have a clear starting and stopping point.D)(All of the above)

D. All of these statements are true.

During the contingent escape condition of a functional analysis, how long should the demand be discontinued after the target response occurs? A)For the rest of the sessionB)For the rest of the dayC)Until the target response occurs againD)Less than one minute

D. When the target response occurs during the contingent escape condition, the demand should be discontinued for about half a minute (not a day or for the rest of the session). "Until the target response occurs again" is incorrect because the demand is delivered again within 60 seconds, typically.

Whenever Pete's grandfather approaches him, Pete engages in self-injurious behavior. Therefore, we can determine the function of Pete's behavior is: A)attention.B)escape.C)automatic.D)(Cannot determine)

D. Just because we know Pete engages in self-injurious behavior when his grandfather approaches him does not mean that we can pinpoint a specific function of the behavior. Does Pete engage in this behavior at any other time? Is he doing it to get his grandfather's attention when he is in close proximity to Pete? Is he doing it to escape the presence of his grandfather (if his grandfather typically leaves him alone when he engages in SIB)? We don't know. Therefore, the answer is "Cannot Determine".

Which of the following is not one of the seven dimensions of applied behavior analysis (Baer, Wolf, & Risley, 1968)? A)AppliedB)BehavioralC)TechnologicalD)Experimental

D. The seven dimensions of applied behavior analysis are: Applied, Behavioral, Analytic, Technological, Conceptual Systems, Effective, and Generality.

Louis is designing an intervention for his client. He makes sure that the techniques making up the intervention are completely identified and described, so that other ABA practitioners may replicate his intervention. Which dimension of behavior analysis (according to Baer, Wolf, & Risley, 1968) is described in this scenario? A)GeneralityB)EffectiveC)BehavioralD)Technological

D. The technological dimension of ABA means that the techniques making up a particular behavioral application are completely identified and described. This means that behavior analysts should write their interventions clearly and concisely so that other ABA practitioners can replicate their intervention.

There are exactly 4 full weeks in February. Mark works Monday through Friday. In February, he missed 2 days of work. Mark showed up for work _______ of the days he was supposed to work in February. A) 98% B) 92.8% C) 60% D) 90%

D. There were 4 full weeks in February. Mark was required to work Monday through Friday (5 days/week). 5 days multiplied by 4 weeks equals 20 days he was supposed to go to work. However, he skipped 2 days of work. 18/20=0.9 0.9x100=90%

On a cumulative record, rapid responding is recorded with a: A)dashed, completely vertical line.B)solid, ascending line.C)solid, descending line.D)dashed, completely horizontal line.

B. On a cumulative record, the solid line will never be completely vertical and will not decrease (descend) since you can't take away a response after it already occurred. The line will not be dashed. A horizontal line would mean there was no responding.

Which measurement procedure is the MOST accurate? A)Frequency recordingB)Whole-Interval recordingC)Partial-Interval recordingD)Momentary Time Sampling

A. Accuracy is the extent to which the observed data indicate what the true values were. If the behavior occurred 3 times, frequency recording of this behavior would be perfectly accurate since it would say exactly how many times the actual behavior occurred. Discontinuous recording procedures (partial-interval, whole-interval, and momentary time sampling) are less accurate than continuous recording procedures (frequency). Discontinuous recording procedures tend to overestimate or underestimate true values of the behavior and are therefore less accurate.

Which verbal operant is a baby likely to learn first? A)A mandB)A tactC)An echoicD)An intraverbal

A. Babies mand in the form of crying for things they need, such as food, a clean diaper, more blankets, attention, etc. Crying is later replaced with more specific mands, such as pointing, vocally manding, etc.

Billy is a behavior analyst. He works with a client who engages is self-injurious behavior. The behavior is very dangerous. Which experimental design(s) would be most appropriate to use in this scenario? A)A "B-A-B" design and an alternating treatments designB)An "A-B-A" design and a "B-A-B-A" designC)A multiple-baseline across settings designD)A changing criterion design

A. These designs would be most beneficial in this scenario because an immediate intervention is necessary. These two experimental designs do not require baseline data to be collected. Multiple baseline designs and A-B-A designs require baselines. Changing criterion designs are also inappropriate because they are used to gradually change behavior over time.

Of the following choices, which includes only examples of time sampling recording procedures? A) Momentary time sampling, partial-interval recording, whole-interval recording B) Rate, duration, frequency C) Partial-interval recording, momentary time sampling, interresponse time D) Momentary time sampling, frequency, latency

A. These three procedures are considered time sampling since you are only trying to gain an accurate sample of the target behavior rather than having to record the actual frequency, duration, rate, etc. of the behavior.

The 'Effective' dimension of Applied Behavior Analysis (Baer, Wolf, & Risley, 1968), states that: A)Procedures developed by behavior analysts must aim to cause a meaningful effect in the client's life.B)All interventions are developed from the basic principles of behavior.C)The behavior changes that are achieved with the intervention should be long-lasting and should generalize to other settings.D)Interventions should demonstrate experimental control over the behavior.

A. "Procedures developed by behavior analysts must aim to cause a meaningful effect in the client's life." This is the definition of the 'effective' dimension of ABA. "Interventions should demonstrate experimental control over the behavior," is the 'analytic' dimension of ABA. "The behavior changes that are achieved with the intervention should be long-lasting and should generalize to other settings," is the 'generality' dimension of ABA. "All interventions are developed from the basic principles of behavior," is the 'conceptual systems' dimension of ABA.

A changing criterion experimental design would be MOST beneficial for: A)helping a client reduce his echolalia.B)teaching a client how to tie his shoes.C)teaching a client how to initiate conversations.D)helping a client reduce his behavior of intentionally setting fires.

A. A changing criterion design is particularly beneficial for decreasing (or increasing) the rate of a behavior over time. However, the target behavior must already be in the learner's repertoire; you would not use a changing criterion design to teach new behaviors. (How are you supposed to increase a behavior that doesn't exist? And how are you supposed to reduce a behavior that doesn't exist?) This design could be used to help a client decrease his frequency of echolalia. A changing criterion design should not be used with extremely dangerous behaviors, such as setting fires. A changing criterion design would allow for the behavior to occur at lower rates over time until the behavior diminishes. Since 0 instances of fire-setting behavior is tolerable, a changing criterion design would not be beneficial. A changing criterion design does not establish/teach new behaviors. It is more beneficial for increasing or decreasing the rates of behavior over time.

When conducting a functional analysis, a behavior analyst should use a(n): A)continuous schedule of reinforcement.B)intermittent schedule of reinforcement.C)reinforcement schedule that matches the natural environment. D)variable ratio schedule of reinforcement.

A. A continuous schedule of reinforcement is also known as an FR1 schedule. It simply means that every instance of the response is reinforced. Since a functional analysis is intended to evoke the target behavior in order to determine which consequence(s) maintain the target behavior, the consequence is typically delivered after every instance of the target behavior. Using intermittent reinforcement (or reinforcement that matches the natural environment) may result in fewer occurrences of the target behavior during the functional analysis. Therefore, behavior analysts typically use continuous schedules of reinforcement during functional analyses.

Which is a function-based operational definition for stealing A) Taking items from a store (or from a person) without their prior permission, notification, or authorization. B) Taking an item and putting it in his pocket when no one is looking. C) Writing fake checks, stealing checkbooks, or stealing someone's credit card number. D) (All of the above)

A. A functional operational definition involves all behaviors that are part of a particular response class. The outcome of the behavior is what is important and not necessarily the topography. Putting items in your pocket when others aren't looking, writing bad checks, stealing checkbooks, and stealing credit card numbers are all examples (topographies) of stealing. However, they are not the only ways to steal something. Therefore, they are considered topographical operational definitions, rather than function-based operational definitions.

What is the MOST accurate way to determine what may function as a reinforcing stimulus for your client? A)A reinforcer assessmentB)A questionnaire with reinforcer optionsC)An open interview with your client where your client can tell you what he/she wants to work forD)A stimulus preference assessment

A. A reinforcer assessment measures what effects particular stimuli have on behavior. If these assessments show that rate of responding increases when a particular stimulus is delivered, it accurately shows that this stimulus would serve as a reinforcer at that time. A stimulus preference assessment determines what MAY function as a reinforcer by seeing what the client prefers. Self-report via interviews and questionnaires are in general much less accurate compared to reinforcer assessments.

A behavior analyst holds up a candy bar and a stuffed animal and says, "Hey Spencer, which one do you want?"What type of procedure did the behavior analyst use in this scenario? A)A forced choice preference assessmentB)A paired stimuli reinforcer assessmentC)A multiple stimuli reinforcer assessmentD)A multiple stimuli preference assessment

A. Because the behavior analyst held up two items and "forced" Spencer to choose one, this is a forced choice preference assessment. A forced choice stimulus preference assessment is also known as a paired stimuli preference assessment. However, "a paired stimuli reinforcer assessment is incorrect. A reinforcer assessment involves conducting an experiment where a person's behavior is monitored to see if a stimulus DOES function as a reinforcer. A stimulus preference assessment helps determine what MAY function as a reinforcer. "A multiple stimuli preference assessment" is incorrect since this refers to 3 or more stimuli being available. A "paired stimuli reinforcer assessment" and a "multiple stimuli reinforcer assessment" do not exist. They are stimulus preference assessments, not reinforcer assessments.

A behavior analyst is writing a treatment program for a client to learn how to use a city bus route. This particular client, however, lives in a rural area with limited access to the city busses. Which dimension of applied behavior analysis did the analyst fail to consider? A)AppliedB)GeneralityC)Technological D)Conceptual Systems

A. Because this client lives in a rural area with limited access to the city bus route, teaching the client to ride the bus will have little impact on their life. The "Effective" dimension of ABA means that procedures developed by behavior analysts must aim to cause a meaningful effect in the client's life.

When a behavior analyst is reviewing a client's medical records, he should look for: A)biological or medical variables that may influence the client's behavior.B)whether or not the client has a diagnosis of Autism. C)how fast the client can run per mile if he were to elope. D)interventions that have been unsuccessful to reduce problem behavior in the past.

A. Behavior analysts review what physiological variables can effect the client's behavior. "Whether or not the client has a diagnosis of Autism." is incorrect because behavior analysts work with all populations. While this may be good information to have, it is not the main reason to review medical records. "Interventions that have been unsuccessful to reduce problem behavior in the past." is incorrect because this information would likely not be revealed in a client's medical record. "How fast the client can run per mile if he were to elope." is incorrect because, while amusing, this information is not likely found in a client's medical record.

This level of scientific study is achieved when the independent variable reliably produces a change in the dependent variable: A)ControlB)DescriptionC)PredictionD)Experimentation

A. Control is achieved when we are able to reliably demonstrate a functional relation, that is, the change in the dependent variable was caused by the independent variable. "Description" is gathering information about observed events. "Prediction" is achieved when two events consistently covary with one another. "Experimentation" is the basic strategy of most sciences.

What is the most accurate measurement system to use when measuring behaviors that have a clear stopping/starting point and each occurrence of the behavior has relatively similar durations? A) Event recording B) Partial-interval recording C) Whole-interval recording D) Momentary time sampling

A. Event recording is most accurate because it involves you recording a check each time the target behavior occurs. This is extremely accurate if the behaviors do not occur so rapidly that it is difficult to accurately measure them, the behavior's duration is relatively similar from one instance to the next, and if the behavior has a clear onset and offset. Time sampling procedures (whole-interval recording, partial-interval recording, and event recording) are not accurate because they do not accurately measure each occurrence of the target behavior.

Which experimental design is most frequently used when conducting a functional analysis? A)An alternating treatments designB)A multiple baseline across settings designC)A multiple baseline across behaviors designD)A reversal design

A. Functional analyses are most commonly conducted with an alternating treatments design. This allows for the behavior analyst to rapidly and randomly alternate between conditions to help determine which condition (function) evokes the target behavior.

A functional analysis reveals that the target behavior happens at relatively high rates in all conditions except the demand condition. What is the most likely function of this problem behavior? A)Automatic reinforcementB)EscapeC)Access to tangiblesD)(All of the above)

A. If a functional analysis reveals that the behavior occurs at relatively the same frequency in all conditions, the behavior is most likely automatically maintained. If a behavior is automatically maintained, a functional analysis may also show lower rates of behavior in the demand condition. This is because the target behavior may be incompatible with the behavior that was demanded.

All of the following are advantages of using single-subject research designs over group-research designs, except: A)There is a control group for comparison that does not receive the independent variable.B)The ability to see individual performance data.C)The ability to see variability and trends in an individual's performance.D)The ability to replicate results.

A. In a single-subject research design, every individual receives the independent variable, there is no control group (this is reserved for group-research designs).

Betsy is assigned a term-paper at the beginning of the semester. She sets aside time each week for an hour to work on the paper, since she knows that it will be a big project, and she wants to earn an 'A.' Betsy turns in her paper at the end of the semester, and when she checks her grades online the next day, she sees that she got a 4.0. What is this an example of? A)Rule-governed behavior.B)Contingency-shaped behavior.C)Variable ratio schedule of reinforcement.D)Positive reinforcement.

A. In this example, Betsy likely had a lot of rules going on (i.e., 'if I work hard now, I won't have to cram later in the semester,' 'if I get a good grade on this assignment, I'll get into that program,' 'my parents won't be disappointed in me,' etc.).

A behavior analyst is tracking how often Janet is verbally aggressive. In a typical week, the behavior analyst observes Janet for 3 hours on Monday, 2 hours on Wednesday, and 1 hour on Friday. What measurement system should the behavior analyst use to track Janet's verbal aggression? A)RateB)FrequencyC)LatencyD)Duration

A. In this example, the behavior analyst wants to determine HOW OFTEN Janet is verbally aggressive. (AKA find out how many times she engages in the behavior.) Frequency data is not appropriate in this scenario since the length of the behavior analyst's observation periods vary from 1 hour to 3 hours. If the behavior analyst used frequency data, it may look like the behavior is decreasing later in the week, even if it is not, because the observation periods are shorter. When observation periods vary significantly in length, a behavior analyst would want to convert frequency to rate by dividing the number of times the behavior occurs across some unit of time. Duration data is not appropriate for this scenario since we are not trying to determine HOW LONG Janet is verbally aggressive. Latency recording is not appropriate in this scenario since this involves measuring the amount of time between the signal for the behavior to start and the behavior starting.

Jesse's teacher says that he keeps acting out in class. The teacher approaches the behavior analyst and explains that Jesse's behavior is likely the result of his diet, that he eats way too much sugar and processed foods, causing problem behaviors. As a behavior analyst, which attitude of science must be considered in this scenario? A)Parsimony.B)Philosophic doubt.C)Empiricism.D)Determinism

A. Parsimony means that the simplest, most logical explanations must be ruled out before more complex explanations are considered. In this example, the teacher should rule out other variables that could be responsible for the problem behavior before making the assumption that something more complex is responsible for controlling the behavior. "Philosophic doubt" is continually questioning the truthfulness of scientific knowledge. "Determinism" is the assumption that the universe is lawful and orderly. "Empiricism" is the objective observation of the phenomena of interest.

A behavior analyst is measuring her client's self-injurious behavior using partial-interval recording. She documents that the behavior occurred 4 times today. However, she believes that the behavior may have occurred more frequently. What can the behavior analyst do, so that in the future the frequency of self-injurious behavior will be more accurate? A)Decrease the length of the intervalsB)Increase the length of the intervalsC)Increase the duration of the behaviorD)Decrease the duration of the behavior

A. Partial-interval recording is conducted by simply marking whether or not the behavior occurred AT ANY TIME during the interval. If the behavior analyst was using a 1-hour interval, she would mark whether or not the behavior occurred during the interval. She would not mark that a behavior occurred many times in an interval. So, if her client was self-injurious 4 times in an interval, it would look like he was only self-injurious once. Shortening the length of the intervals will help gain a more accurate depiction of the target behavior. If the behavior analyst used 15 minute intervals, there is a good chance that she would have been able to document all 4 instances of the target behavior.

Partial-interval recording tends to _________ the duration of a behavior. A) overestimate B) underestimate C) accurately measure D) (None of the above)

A. Partial-interval recording typically overestimates a behavior's duration. Consider this: you are monitoring whether or not your client stays inside his group home, rather than eloping. You monitor the behavior for an hour. The hour is broken up into 10 intervals. If he is inside his home for any amount time within an interval, it is marked as a successful interval. If your client eloped every single interval during that hour (but he was present at some point during that interval), it would look like he was present 100% of the time that interval. Time sampling procedures, such as partial-interval recording, are not necessarily accurate. Accuracy refers to the true value of a certain behavior. Time sampling is simply a momentary point in time where you monitor the behavior. It isn't accurate because the behavior likely occurs when you are not monitoring as well.

Sal went to a coffee shop on Monday, Wednesday, and Thursday this week. What is the frequency of his behavior? A) 3 instances B) 3 instances per week C) 0.42 instances per day D) 42%

A. Sal went to the coffee shop 3 times. Frequency is a process of simply tallying how many times the behavior occurs. Therefore, the answer is simply "3 instances". "42% is incorrect since frequency is not a percentage. "3 instances per week" and "0.42 instances per day" are also incorrect since they are rates. (Rates are frequency measures, divided by some unit of time.) For the exam, frequency is simply the number of times a particular behavior occurs.

Samantha's son went off to college earlier this year. Because he is so busy, he only calls her a couple times per month on average. Samantha wishes her son would call her more frequently. Which recording method would be best for Samantha to use to track how often her son calls her? A) Frequency recording B) Duration recording C) Whole-Interval recording D) Momentary Time Sampling

A. Samantha wants to track how often her son calls her. Since the rate is so low right now (2x/month), she can easily and accurately track the amount of times he calls her by using this tally method. This is the most practical, accurate, and easy answer option. "Whole-interval recording" is incorrect because this involves recording whether the behavior of calling his mother occurs throughout an entire interval. Since phone calls only happen a couple times per month, this is not the best answer. "Momentary time sampling" is also incorrect because the likelihood of him calling her at the exact time she is recording data is unlikely. "Duration recording" is incorrect because it involves recording how long a behavior takes place. If she wanted to increase the amount of time they spoke together when he did call, then duration recording would be appropriate. Duration recording does not record the time between one phone call and the next phone call. This would be inter-response time.

Louis learns how to use a microwave at his school. He now cooks his own lunch every day in the microwave at his school. However, he never uses the microwave at home when his parents ask him to do so. This is most likely due to: A)Inadequate stimulus generalizationB)Inadequate response generalizationC)Inadequate stimulus discriminationD)Inadequate response discrimination

A. Stimulus generalization involves bringing one response under control of more stimuli (he should use the microwave at school and use the microwave at his home). Since the stimuli at his home are not evoking the desired response, this hints that there is not adequate stimulus generalization. "Response generalization" is incorrect since this would mean that he was learning new behaviors that resulted in the same reinforcer (such as cooking food on the stove or in the oven). "Stimulus discrimination" is incorrect since this would involve evoking a response under fewer stimuli. "Response discrimination" is not necessarily a behavior-analytic term but it would involve fewer responses and we are looking for generalization in this scenario.

How long should a behavior analyst need to keep clients' records? A) 7 years B) 7 months C) Until the behavior analyst no longer works with that client D) 7 years OR until the records are handed over to the next behavior analyst who will be working with this client

A. The BACB® states that records must be maintained for 7 years from when services were terminated.

Which dimension of applied behavior analysis means that interventions should demonstrate experimental control over the behavior? A)AnalyticB)EffectiveC)Conceptually SystematicD)Technological

A. The analytic dimension of applied behavior analysis means that interventions should demonstrate experimental control over the behavior (in other words, the intervention is the cause of the behavior change).

Tatiana is a behavior analyst who works with children. She receives a referral to work with an adult, but has never worked with an adult before. Of the following options, what is the most appropriate course of action she could take? A)Politely decline the referral.B)Take on the client. Conduct an FA and write the intervention. Then ensure a behavior analyst who is familiar working with adults reviews her plan prior to implementation.C)Refer the client to a behavior analyst who is familiar working with adults and supervise that behavior analyst.D)Contact the BACB® to file a complaint.

A. The best option in this scenario would be to politely decline. If Tatiana declines the referral, she should make recommendations of behavior analysts nearby who have the necessary experience to take on the case. Tatiana would not need to supervise that behavior analyst (if anything she may shadow the analyst to gain competency in working with that population). Since she has never worked with this population, conducting a functional analysis and writing the intervention plan is not recommended. This would not warrant filing a complaint to the BACB® since no violation has been committed.

Which dimension of applied behavior analysis (Baer, Wolf, & Risley, 1968) means that all interventions are developed from the basic principles of behavior? A)Conceptual SystemsB)BehavioralC)AnalyticD)Technological

A. The conceptual systems dimension of ABA means that all interventions are developed from the basic principles of behavior. This means that behavior analysts do not use interventions from other professional fields. We also only use behavior-analytic interventions that are proven through scientific research in the field of ABA.

Phil works at a factory. He assembles cardboard boxes for a living. He has to fold the box up correctly and then tape it together. Today, his boss said, "For every blue box you assemble, you will receive $2. For every red box you assemble, you will receive $1." It takes the same time and effort to assemble the red and blue boxes. Phil assembles 40 blue boxes today. According to the Matching Law, Phil likely also assembled _______________ red boxes today. A)0 B)40 C)20 D)80

A. The matching law refers to the distribution of responses when there is more than one schedule of reinforcement available at the same time. With concurrent variable interval or concurrent fixed interval schedules of reinforcement, the person is likely to distribute their responding proportionally between the two schedules of reinforcement.

Which scenario would warrant a termination of services for your client? A) Intervention goals are met and no other behavioral concerns are apparent at this time. B) You do not speak the language that the client and his family speak but a translator is available. C) Intervention goals have not been met within 3 months. D) You are competent and trained to work with your client who is an adult, but recently decided to only work with children.

A. The ultimate goal of providing behavior-analytic services is to allow people to live the most independent and successful lives possible. Therefore, it is appropriate (and necessary) to terminate a client who has met all of his intervention goals and there are no other behavioral concerns at that time. You speaking a language that is different from your client does not warrant termination of services since there is a translator available (though it may benefit the client more if there is a competent behavior analyst who speaks the same language as the client). "Intervention goals have not been met within 3 months" does not warrant termination of services necessarily since some interventions take longer than others, and it does not say how long the original intervention goals were supposed to take. Finally, simply deciding that you no longer want to work with adults does not warrant termination of services. It would be necessary for you to continue to provide services until another behavior analyst can take on the case.

Under which circumstance(s) could a behavior analyst reveal the identity of their client, even if the client does not consent? A) If the client's records are subpoenaed to court. B) If your client threatens to run away. C) If your lawyer requests that his identification be revealed. D) (All of the above)

A. There are only a few instances where a behavior analyst could reveal confidential information about their client without the client's (or their guardian's) consent. A behavior analyst may reveal a client's identity if he/she truly believes the client may injure or kill someone. (Threatening to run away would likely not warrant the revealing of their identity. However, if the client DID run away and they were in imminent danger, this could be appropriate.) A behavior analyst must reveal a client's identity if their records are subpoenaed to court. (If a lawyer asks for a client's identity without a subpoena, it is not appropriate to reveal their identity.) Finally, if a behavior analyst is seeking other professional services for the client, it may be necessary to disclose their identity.

You are visiting an old friend, who is also a behavior analyst. He is conducting research in an attempt to discover a new intervention technique to use with people who are diagnosed with Prader-Willi Syndrome. He asks you to take some treatment integrity data on his supervisees. You are trained at collecting treatment integrity data. However, you are not trained in working with individuals who are diagnosed with Prader-Willi Syndrome. Could you collect treatment integrity data in this situation? A)YesB)No. Since you have never worked with this population, you would want to gain the relevant training and experience first.C)Yes. But your friend (who is a behavior analyst) would have to supervise you.D)No. Behavior analysts do not work with this population.

A. This is a tricky one. But since you are trained on collecting treatment integrity data, it would be appropriate for you to collect it in this scenario. Since you are a behavior analyst who is already trained in this skill, your friend would likely not have to supervise you first. You would not necessarily need more training and expertise on Prader-Willi Syndrome. You are simply collecting treatment integrity data on the supervisees' behavior. And since you are competent in doing so, it would be appropriate in this situation. Behavior analysts work with all populations, including Prader-Willi Syndrome.

Which of the following is considered a private event? A)Sore muscles after an intense workoutB)Practicing your solo for a choir concertC)Texting your friend about lunch plansD)Taking your dog outside in the backyard

A. This is an example of a private event. The other examples, "practicing your solo for a choir concert," "texting your friend about lunch plans," and "taking your dog outside in the backyard," are not considered private events.

Tim gets home from school and is hungry. He opens up the cupboard and grabs a snack and starts eating it. This is an example of a(n): A)Automatic reinforcement contingency.B)Socially-mediated reinforcement contingency.C)Automatic punishment contingency. D)Socially-mediated punishment contingency.

A. This is an example of an automatic contingency because Tim was able to get a snack on his own, and he didn't need any other person to deliver the reinforcement. He was able to open the cupboard and get a snack. Therefore, this is automatic reinforcement. If he had to ask his parent to get him a snack, then it would be considered a socially-mediated contingency.

Before the week of Christmas, the family that Tina supervises for says they want to get her something to thank her for all the work that she has done over the past year. The family says they won't take no for an answer. What should Tina do in this scenario? A) Explain that the gift must be less than $10 and cannot be a regular occurrence. B) Give the family your Amazon® list of items. C) Tell the family they can take you out for a nice meal. D) Call the agency and explain the situation and have them reach out to the family about the gift giving policy.

A. This one is a little tricky, but according to the BACB® ethics code, behavior analysts do not give gifts to or accept gifts from clients, stakeholders, supervisees, or trainees with a monetary value of more than $10 US dollars. Behavior analysts make clients and stakeholders aware of this requirement at the onset of the professional relationship. A gift is acceptable if it functions as an infrequent expression of gratitude and does not result in financial benefit to the recipient. Instances of giving or accepting ongoing or cumulative gifts may rise to the level of a violation of this standard if the gifts become a regularly expected source of income or value to the recipient. "Give the family your Amazon® list of items," "Tell the family they can take you out for a nice meal," and "Call the agency and explain the situation and have them reach out to the family about the gift giving policy" are incorrect because they go against the ethical code.

Joique hates loud noises. His mother is using a systematic desensitization program to help him tolerate these loud noises. She sits in a small room with Joique. Every 15 seconds, she loudly bangs pots and pans together. She wants him to stay in the room as long as he can, rather than running out of the room. Which unit of measurement would be best for determining whether or not Joique is increasing his tolerance to loud noises? A) The duration of Joique staying in the room with his mother B) The interresponse time of Joique banging the pots and pans C) The frequency of Joique's behavior of running out of the room D) The latency of Joique's behavior of running out of the room

A. This one is pretty straightforward; don't think too much into it. Joique's mother is trying to increase the amount of time Joique stays in the loud room. Therefore, simply recording how long he stays in the room (i.e. duration) is likely the best measurement system to use. "The interresponse time of Joique banging the pots and pans" is incorrect since his mom was the one doing this. "The frequency of Joique's behavior of running out of the room" is less correct. Since the length of sessions may vary drastically, this is not the best unit of measurement. For example, if he only ran out of the room 1 time in a day, this would look favorable. However, if the session was only 15 seconds long, this would not show an improvement to tolerating loud noises. Finally, "The latency of Joique's behavior of running out of the room" is less correct. Since his mother bangs the pots and pans every 15 seconds, it is likely that he will eventually not run out of the room within 15 seconds of her banging the pots and pans. Therefore, if we were measuring latency, that interval would be voided. If he ran out of the room only 1 time in an hour session, the latency for that day would be the amount of time between the bang and running out of the room... which would inaccurately show that he was not improving his tolerance to these noises.

Behaviors that were established using a behavioral intervention are considered ____________ once they allow the learner to appropriately respond to new situations. A)generativeB)maintainedC)functionalD)topographical

A. When a behavior that is taught allows the learner to appropriately respond to untaught situations, the behavior is considered generative. "Topographical" is incorrect. The topography of a behavior simply refers to what the behavior looks like, which doesn't apply to this situation. "Functional" is less correct. A behavior that is taught in a training environment may be intended to only serve one function. Therefore, the taught behavior would still be functional even if it did not occur under new stimuli. "Maintained" is less correct. Maintenance refers to the behavior continuing to occur after training has discontinued. However, it does not necessarily mean that the learner will use behavior to untaught stimuli.

A behavior analyst is visually analyzing a graph. He notices that during the baseline phase, the behavior occurred exactly 5 times per session. There were 10 baseline sessions. The behavioral data in the baseline session would have: A) a zero trend. B) a decreasing trend. C) an increasing trend. D) a positive trend.

A. When analyzing the trend (the general overall direction of the data path), zero trend means that the data is neither increasing or decreasing. Note: A behavior can still have "zero trend" even when the behavior occurred 5 times per session. Zero trend does not mean the data is at 0, it simply means there is no clear trend in the data because it is stable.

A behavior analyst is called into a classroom to work with Suzy. Suzy's teacher complains to the behavior analyst that Suzy is lazy. When the teacher tells the students to start an assignment, it generally takes about 5 minutes for Suzy to comply. When she starts the assignment, she is on task the entire time. The behavior analyst should work with Suzy on: A) decreasing the latency of her response. B) decreasing the duration of her response. C) decreasing the likelihood of an extinction burst. D) decreasing the interresponse time of her response

A. When the opportunity to respond starts, it takes Suzy 5 minutes to respond. Therefore, the behavior analyst would want to decrease this amount of time (AKA decrease the latency). Since she is on task the entire time when she does begin working, and the duration of the response was not mentioned, "decreasing the duration of her response" is incorrect. "Decreasing the likelihood of an extinction burst" is incorrect since an extinction burst wasn't mentioned in this scenario. Finally, "Decreasing the interresponse time of her response" is less correct. The opportunity to respond is contingent on the teacher's instructions. If the teacher delivered instructions less frequently one day, Suzy's interresponse time would not need to be decreased.

Which behavior is most likely to be tracked using event recording? A)How many meals a client eats in one dayB)How many times a client blinks in one dayC)How many hours a client sleeps in one dayD)(All of the above)

A. When using event recording, the behavior should not occur so often that it would be nearly impossible to collect data accurately. Since blinking occurs so frequently, this answer is incorrect. "How many hours a client sleeps in one day" is incorrect since duration would be used to record this (non)behavior. A data collector could easily and accurately count and record how many times a client eats a meal. Therefore, this is the correct answer.

A behavior analyst wants to determine what will serve as an effective punisher for his client. He has 5 possible stimuli that may serve as punishers. Contingent upon a target response, the behavior analyst delivers a punisher and measures the amount of time it took to suppress that response. He tries this with all 5 possible punishers and determines that one particular punisher is more effective at suppressing the response than the other punishers. What kind of procedure did the behavior analyst conduct in this scenario? A)A stimulus avoidance punisher assessmentB)A brief punisher assessmentC)An activity punisher assessmentD)A choice punisher assessment

B. A brief punisher assessment determines which available punisher is most effective. The available punishers are presented in a multi-element design on a target behavior, and the suppression of the behavior is measured. The punisher that is most effective in suppressing the behavior is the most potent punisher. "A choice punisher assessment" is incorrect since this would simply involve asking the client (or those who know him well) what would likely be the most effective punisher. "An activity punisher assessment" is incorrect since this would involve having several activities freely available and the one that is avoided may possibly serve as a punisher. "A stimulus avoidance punisher assessment" is also incorrect since this would involve non-contingently presenting possible punishers to the client and taking note of which ones he avoids.

Lexi says, "It never fails. Whenever there is a full moon, people start acting weird." However, Lexi has no data to support her theory. Which scientific analysis is Lexi using? A)A Descriptive AnalysisB)A Hypothetical AnalysisC)An Experimental AnalysisD)A Correlational Analysis

B. A hypothetical analysis is simply assuming that two things go together, without taking any data or manipulating any variables. Lexi did not take data on the full moon or "weird behavior" and did not manipulate these variables. "Experimental analysis" is incorrect since this would require manipulating the environment and recording the behavior. "Correlation(al) analysis" and "descriptive analysis" are incorrect since they would involve recording the behavior as well.

Always write treatment records as if they were: A) going to be included in a novel you may write at a later time. B) going to be evidence in a court case. C) going to be read by the client's family. D) going to be used to write other clients' treatment records.

B. A judge could subpoena you to court regarding your client and the services you rendered. Therefore, all files and documents should be written as if they were going to be used in a court hearing (as they in fact may at some point in time). "Going to be read by the client's family" is incorrect since we do not know with the information given if the client is affiliated with his family anymore. You would not use records from one client to write records regarding another client of yours. Keeping client-specific records to use as a reference when writing your novel is not correct.

Your friend asks you if you want to hang out. You ask her if she wants to go to the beach or go to the movies. Which type of preference assessment did you conduct? A)A paired-stimuli reinforcer assessmentB)A paired-stimuli preference assessmentC)A forced-choice reinforcer assessmentD)A forced-choice reinforcer assessment without replacement

B. A paired-stimuli preference assessment involves presenting two stimuli and "forcing" the person to choose an item. A paired-stimuli preference assessment is also referred to as a "forced-choice" stimulus preference assessment. This example is NOT a reinforcer assessment; it simply involved you asking your friend what she preferred and did not involve conducting an experiment to determine whether the stimulus would function as a reinforcer.

A father asks his daughter if she wants spaghetti or pizza for dinner, while holding each item up to show her. His daughter chooses pizza. Which type of preference assessment did the father conduct? A)A paired-stimuli reinforcer assessmentB)A paired-stimuli preference assessmentC)A forced-choice reinforcer assessmentD)A forced-choice reinforcer assessment without replacement

B. A paired-stimuli preference assessment involves presenting two stimuli and "forcing" the person to choose the higher preferred item. A paired-stimuli preference assessment is also referred to as a "forced-choice" stimulus preference assessment. This example is NOT a reinforcer assessment; it simply involved asking the person what they preferred and did not involve conducting an experiment to determine whether the stimulus would function as a reinforcer.

Which type of analysis is used to determine the value of the independent variable needed to change behavior? A)A component analysisB)A parametric analysisC)A comparative analysisD)Behavior analysis

B. A parametric analysis is used to determine what amount of the independent variable is necessary to cause the most effective behavior change. For example, you could run a parametric analysis to see if a 1-minute timeout, 5-minute timeout, or 10-minute timeout is most effective for decreasing a child's behavior of fighting with his sibling. "A component analysis" is incorrect, since this involves analyzing which components of a treatment package are responsible for the behavior change. "A comparative analysis" and "behavior analysis" are not used to determine what amount of an independent variable is necessary to change the behavior.

A behavior analyst is working with a child who is in school. However, the principal does not want the behavior analyst to observe the child during class because she believes this will distract all of the students. Also, the teacher does not have time to collect data herself. Which functional behavior assessment tool could the behavior analyst use in this situation? A)ABC continuous recordingB)A questionnaire or rating scaleC)ABC narrative recordingD)A brief functional analysis

B. A questionnaire or rating scale could be given to the teacher before or after school and does not involve directly observing the child. ABC recording procedures involve directly recording the behavior. Since the behavior analyst is not allowed to observe during school and the teacher does not have time to fill out the datasheets, these procedures would not work. Finally, a brief functional analysis would still involve directly working with the child in the classroom, so this option is also incorrect.

Juri is at an Easter egg hunt with several other children. The goal of the Easter egg hunt is to collect as many eggs as possible. When an adult says, "Ready, Set, Go!" Juri collects as many eggs as he can. What unit of measure would be best for determining how many eggs Juri collects? A)DurationB)FrequencyC)RateD)Interresponse time

B. A simple frequency measure (AKA counting) would be best to determine how many eggs Juri collected. "Duration" is incorrect since this would measure the amount of time it took him to pick up an egg. "Rate" is incorrect since this involves how many eggs he collected over a specific span of time (i.e. 3 eggs per minute). Since we do not know how long he collected eggs for, the rate alone could not tell us how many total eggs he collected. "Interresponse time" is incorrect since this would measure the amount of time between him collecting one egg from the next, which would not help us determine the total amount of eggs he collected.

You are working at an ABA company and a perspective parent contacts you to see if you'll intervene on their teenage child's cursing behavior. You conduct an assessment and find out that the cursing behavior rarely occurs, and typically only occurs when he is alone playing video games. You determine that his cursing is at the same rate, or even lower, than a typical teenager. What should you do in this scenario? A)Instruct the parent to start a differential reinforcement of other behavior procedure. B)You determine that behavior analytic services are not necessary at this time.C)Tell the parent to start taking baseline data so that you can begin working on a successful intervention strategy.D)Refer the parent to a psychologist that specializes in oppositional defiant disorder.

B. After talking with the parent and gathering more information about the behavior of concern, you determine that behavior analytic services are not needed at this time. The behavior of concern does not happen frequently enough or at a higher rate than an average teenager, and it does not require intervention. "Instruct the parent to start a differential reinforcement of other behavior procedure," "Tell the parent to start taking baseline data so that you can begin working on a successful intervention strategy," and, "Refer the parent to a psychologist that specializes in oppositional defiant disorder," are all incorrect, since at this time, behavior analytic services are unwarranted.

A baseline phase: A)occurs before the experimenter starts data collection.B)is not necessary for alternating treatment designs.C)is not necessary for reversal designs.D)(All of the above)

B. Alternating treatments designs typically include a baseline. However, they aren't mandatory. The experimenter DOES collect data during the baseline phase. Reversal designs do require baseline phases (the reversal of the treatment means the experiment returns to a baseline phase).

A behavior analyst is working with a client who swallows large items such as rocks, pencils, and cotton balls. Which experimental design(s) would be recommended for this target behavior? A)A reversal designB)An alternating treatments designC)(Both A and B)D)(Neither A nor B)

B. Because this behavior is so dangerous, collecting baseline data is not recommended. A behavior analyst would not want to sit around watching how many rocks his client swallows to gain a baseline. With an alternating treatments experimental design, a baseline is not always necessary. With a reversal design, a baseline is necessary.

Marjorie has finally finished up with the last of her supervision requirements. She has been documenting all of her meetings and hours with her supervisor, Kane. What should Kane tell Marjorie to do with the records of her supervision? A)Mail them to the BACB®.B)Retain them for at least 7 years.C)Burn them in a celebration because you finished.D)Store them at the location of your practicum.

B. Behavior analysts must retain their supervision documentation for at least 7 years and as otherwise required by law and other relevant parties and instruct their supervisees or trainees to do the same.

For which condition of a functional analysis would the experimenter withhold attention after the target response occurs? A)The attention conditionB)The demand conditionC)The tangibles conditionD)(All of the above)

B. Contingent upon the target behavior occurring during the demand condition of a functional analysis, the experimenter removes the materials used for that demand and also removes attention. This condition helps determine if the client is engaging in the target response to "get out of" what is being asked of him. Attention should be minimized during the demand phase to help isolate escape as a function (as opposed to access to attention). "The attention condition" is incorrect since this involves providing attention when the behavior occurs, not withholding it. "The tangibles condition" is also incorrect since this involves providing a tangible item to the client when he/she engages in the targeted response, which in turn provides attention.

With regards to behavior analysis, ___________ is the philosophical assumption that all behavior is lawful and orderly. A)SelectionismB)DeterminismC)ParsimonyD)Empiricism

B. Determinism is the philosophical assumption that all behavior is lawful and orderly. Selectionism is the assumption that all behavior is a result of interactions with the environment. Parsimony is using the simplest and most logical explanations to explain why a behavior is occurring. Empiricism is the practice of objective observation of the behavior of interest.

How can we tell that a functional relation exists? A)When the dependent variable reliably produces a change in the independent variable.B)When the independent variable reliably produces a change in the dependent variable.C)When the behavior drops to levels above or below the expected baseline value.D)When the independent variable correlates with the dependent variable.

B. The dependent variable is the measure of the behavior, and the independent variable is the variable that is added that changes the dependent variable. A functional relation exists when a well-controlled experiment demonstrates that a change in the dependent variable is reliably produced by specific manipulations of the independent variable).

This philosophical assumption of science states that scientific knowledge is built on the practice of objective observation of the phenomena of interest: A)Parsimony.B)Empiricism.C)Selectionism.D)Determinism.

B. Empiricism means that we observe behaviors objectively and do not make any assumptions about why the behavior is occurring. "Determinism" is incorrect because it is the general assumption that the entire universe is orderly and that all phenomena occur as the result of some other event. "Parsimony" is using the simplest and most logical explanations to explain why a behavior is occurring. "Selectionism" is incorrect because it is the philosophical assumption that all behavior, in all living organisms, is the result of an interaction with the environment.

A BCBA® who just passed the certification exam is assigned her first client. The client is a child with autism who engages in self-injurious behavior. The BCBA® previously worked with children with autism who engaged in self-injurious behavior during her internship. She also wrote her thesis on self-injurious behavior. Does this newly certified BCBA® have the relevant experience to work with this client? A)No, she should not take on this client. She has never had paid experience working with this population. B)Yes, she likely has the relevant education and experience to take on this client. If she ever feels unprepared, she should reach out to a BCBA® who has more relevant experience in this area.C)No, she should not take on this client. Her experience of working with this population and writing her thesis on her experience is not adequate.D)No, she must be supervised by a BCBA at all times for 1 year until she can ensure her competence in this area.

B. Given that this BCBA® has relevant training and experience working with this population, and that she has conducted research with this population, she likely has the relevant education and experience to take on this case. If she ever feels incompetent, she should reach out to a BCBA® who has more experience working with this population.

Rand is trying to increase the amount of time he holds his breath under water. On his first attempt, he held his breath for 60 seconds. On his second attempt, he held his breath for 70 seconds. On his third attempt, he held his breath for 40 seconds. What is the average duration of his breath-holding behavior? A)170 secondsB)56.6 secondsC)0.01 secondsD)66.7 seconds

B. He held his breath 3 times (once for 60 seconds, once for 70 seconds, and once for 40 seconds). (60+70+40)/3=56.6 seconds per attempt. When determining the average duration, add up all of the sums and divide it by the number of total attempts. All of the other answer choices are mathematically incorrect.

It would be appropriate to conduct a functional analysis for which of the following behavior(s)? A)Head banging that results in bumps and bruisesB)TantrumsC)Stealing from stores D)(All of the above)

B. It would be beneficial to use a functional analysis to determine the function of a tantrum. However, there are several reasons a behavior analyst would not want to conduct a functional analysis. The first reason is if the behavior is severely dangerous. A functional analysis intends to purposefully evoke the target behavior. Behavior that cannot be tolerated at all (such as head banging that is causing great bodily harm) should not be assessed with a functional assessment. If a behavior is illegal and could result in the client being arrested, a behavior analyst would likely not want to purposefully evoke this behavior by using a functional analysis.

A captain of a ship sees that his ship is heading straight towards an iceberg. It takes him 2 minutes to start turning the steering wheel of the ship and it takes him 2 seconds to turn the steering wheel completely to the left. The ship barely misses the iceberg. To avoid near disasters like this in the future, the captain likely needs to: A) decrease the duration of turning the steering wheel. Correct answer B) decrease the latency of turning the steering wheel. C) increase the latency of turning the steering wheel. D) decrease the interresponse time of getting an eye exam.

B. Latency is the amount of time between the signal that is intended to evoke the behavior and the response occurring. The signal (the iceberg) indicates that a certain behavior (turning the ship) should be initiated. Since the captain waited 2 minutes to begin turning the steering wheel, he should try to do this quicker in the future. "Increase the latency of turning the steering wheel" is incorrect because this would mean that he would wait longer to turn the steering wheel. "Decrease the duration of turning the steering wheel" is less correct. He turned the steering wheel in 2 seconds. While he may be able to turn it a little faster, he would likely benefit by not waiting so long to turn it in the first place. "Decrease the interresponse time of getting an eye exam" is comical, but incorrect with the information given.

__________ is a subfield of behavior analysis that focuses on analyzing the principles of behavior, while __________ is a subfield of behavior analysis that focuses on socially significant problem solving using the principles of behavior. A)Applied Behavior Analysis; Experimental Analysis of BehaviorB)Experimental Analysis of Behavior; Applied Behavior AnalysisC)Methodological Behaviorism; Applied Behavior AnalysisD)Experimental Analysis of Behavior; Radical Behaviorism

B. Methodological behaviorism is an approach that restricts the science of psychology to only those independent and dependent variables that two independent people can directly observe, and radical behaviorism is an approach that address all psychology in terms of the principles of behavior.

A methodological behaviorist would not consider which of the following events a behavior? A)Mary slaps Kelly on the face.B)Kelly avoids Mary because she is scared.C)Mary follows Kelly around at recess.D)Kelly tells the teacher about Mary's behavior.

B. Methodological behaviorists do not consider private events as behaviors. The other examples, "Mary follows Kelly around at recess," "Kelly tells the teacher about Mary's behavior," and, "Mary slaps Kelly on the face," are examples of public events that methodological behaviorists would consider a behavior.

The last 5 data points on a cumulative record are all at 20 occurrences of a target behavior. What does this data indicate? A)Stable respondingB)No respondingC)An increase in respondingD)A decrease in responding

B. On a cumulative record, if the data line is horizontal it indicates that there were no responses during this period. "Stable responding" is incorrect since the subject is not responding at all.

Of the following statements, which is NOT true regarding alternating treatments experimental designs? A)The behavior analyst should make each condition highly discriminable from the other conditions.B)A stable baseline is necessary before starting the intervention.C)Experimental control is established when one condition is consistently higher than another condition.D)The conditions of the experiment are altered rapidly.

B. One of the benefits of an alternating treatments design is that the baseline does not necessarily have to be stable prior to starting the intervention. In fact, you don't necessarily need a baseline. This is beneficial when working on behaviors that are severely dangerous, since collecting baseline data on such behaviors exposes the client to further risk of injury or death. In an alternating treatments design, the experimental conditions are rapidly changing. Changes from one condition to the next can occur every session, or even in the middle of a session. Conditions should be highly discriminable from one another so that the client can more easily differentiate between conditions. When the data from 2 or more conditions are consistently different from one another (i.e. always higher or lower), experimental control has been established.

Partial-interval recording tends to _________ the frequency/rate of a behavior. A)overestimateB)underestimateC)accurately measureD)(None of the above)

B. Partial-interval recording typically underestimates the frequency/rate of a behavior. Consider this: you are monitoring how many times your client spits on the floor of his group home. You monitor the behavior using partial-interval recording with 10-minute intervals. If he spits on the floor at any time during that interval, then the interval is marked "+". If he spits on the floor 29 times in that interval, the interval is still marked "+". Therefore, partial-interval recording can underestimate the rate of a behavior if it occurs more than one time per interval. Time sampling procedures, such as partial-interval recording, are not necessarily accurate. Accuracy refers to the true value of a certain behavior; it is a true statement about how much of the behavior occurred. Since partial-interval recording does not capture every instance of the target behavior, it is not a true and accurate measure of the target behavior. Although time sampling procedures are less accurate than continuous data collection methods, they are sometimes preferable due to resource constraints.

Doug got stung by a bee when he was 6 years old on the playground, and it caused him a lot of pain. Now, even 10 years later, Doug is still scared of bees. His heart starts racing even when he is near a bee. This is an example of: A)Operant conditioningB)Respondent conditioningC)Operant extinctionD)Respondent extinction

B. Respondent conditioning is the process where a neutral stimulus acquires the eliciting properties of an unconditioned stimulus through pairing of the neutral and unconditioned stimulus. The previously neutral stimulus (bee) now elicits the conditioned response (heart racing).

Gilligan enjoyed fishing ever since his father taught him how to fish. He loved trying out new fishing poles and he would often spend nearly half of his paycheck on fishing supplies. He used to catch fish nearly every day.One day, Gilligan was on a cruise boat. The boat sank and he was forced to swim to a nearby deserted island. He did not have a fishing pole with him. Therefore, he created a fishing net out of tree roots to catch fish. This allowed for him to eat until he was finally rescued.Gilligan fishing with a net is an example of: A)stimulus generalization.B)response generalization.C)response maintenance.D)(None of the above)

B. Response generalization occurs when someone engages in a new response that is functionally equivalent to a trained response. Gilligan engaged in a new response (fishing with a net), rather than the trained response (fishing with a fishing pole), to gain the same reinforcer (fish). Stimulus generalization is incorrect since this means that the same response was evoked by a new stimulus. Response maintenance is incorrect since this means that a trained response continued to occur after training concluded. This example did not say that Gilligan was ever taught to fish with a net.

You are a behavior analyst who has recently been assigned to work with Harry. Historically, candy has been an effective reinforcer for Harry. Over the past week, whenever Harry takes a bite of any kind of candy he cries and spits it out. What should you do in this situation? A)Conduct a preference assessment to see which candy Harry prefers. B)Review Harry's dentistry records to see when he had his last checkup and make a referral to his dentist if necessary.C)Tell Harry's parents to stop giving him candy. D)Continue to give Harry the candy using an escape-extinction procedure.

B. Since Harry typically enjoys eating candy, this may be indicative of a medical issue, such as a cavity. Therefore, reviewing his dentistry records and making a referral to his dentist is likely the best course of action for this scenario. The behavior analyst should not force Harry to eat candy if he does not want to eat it. "Conduct a preference assessment to see which candy Harry prefers." is incorrect since there may be an underlying medical condition that is causing him pain when he eats sugary foods. "Tell Harry's parents to stop giving him candy." is incorrect because, as a behavior analyst, we don't want to give advice outside our scope of practice. "Continue to give Harry the candy using an escape-extinction procedure." is incorrect because this could be considered unethical by forcing the client to continue to eat, and could worsen the potential medical condition.

A behavior analyst is working with a child named Kerry. Kerry requests to use the bathroom several times, but the behavior analyst tells her to wait. The behavior analyst knows that if he makes a bathroom break contingent upon the completion of a difficult task, she is more likely to complete that task.What is wrong with this scenario? A)Using negative reinforcement is unethical.B)Creating a strong establishing operation by denying basic human rights is unethical.C)Kerry will likely use bathroom breaks to escape tasks in the future.D)(None of the above)

B. The behavior analyst should not have denied Kerry's basic human right of going to the bathroom. (If there was a history of escape-maintained bathroom breaks and she went to the bathroom shortly before she asked to use the bathroom, an escape extinction procedure may be ethical.) Since we do not know if she has a history of escaping task demands by going to the restroom, that answer choice is incorrect. Using negative reinforcement procedures is not necessarily unethical if you are not denying basic human rights.

Andrew, a behavior analyst, recently began working with a client named Bryce. Bryce had a behavior analyst in the past, but she moved out of state about a year ago. Andrew reviews all the documents that were left behind by the previous behavior analyst and notices that a token economy procedure was very effective at decreasing Bryce's elopement behavior in the past. In the past several months, Bryce has been eloping frequently. A)Andrew should use a token economy procedure to decrease Bryce's elopement.B)Andrew should conduct a functional behavior assessment on Bryce's elopement.C)Andrew should not use a token economy because elopement should not be rewarded. D)(None of the above)

B. The function of a behavior can change over time. Therefore, Andrew should confirm the function of the behavior prior to implementing an intervention. Also, just because an intervention was effective in the past does not mean that it will be effective now. Saying Andrew "should use a token economy" is not confirmed at this time. "Andrew should not use a token economy because elopement should not be rewarded" is incorrect with the information given. The example says that a token economy WAS effective in the past.

You are conducting an experiment with your client to reduce vocal stereotypy by using a treatment package that consists of a response-interruption and redirection procedure (RIRD) and differential reinforcement of other behaviors (DRO) procedure. Baseline rates of vocal stereotypy are taken, and then the treatment package is implemented. In this experiment, the treatment package used was the __________ variable and the rate of vocal stereotypy is the __________ variable. A)dependent; independentB)independent; dependentC)dependent; co-independentD)co-dependent; independent

B. The independent variable is the variable the experimenter systematically manipulates (in this case, the treatment package), and the dependent variable is the measure of the subject's behavior (in this case, the vocal stereotypy).

A multiple probe design may be preferred over a traditional multiple baseline design because: A)it can be used across multiple behaviors, subjects, and settings.B)baseline data does not have to be continuously collected.C)a stable baseline is not necessary.D)(All of the above)

B. The major advantage of a multiple probe design is that the experimenter does not have to continuously collect baseline data. They simply collect probe data 1-3 times per step of the task that is being taught. A stable baseline is necessary. Multiple baseline designs can also be used across behaviors, settings, and subjects, and therefore this is not considered an advantage of multiple probe designs.

A behavior analyst observes her client while he is at work. Her client's guardian reports that he sometimes refuses to work if he does not get his way. The behavior analyst observes him from a distance and he does not know that she is watching him work (although the client's guardian does know that the behavior analyst is observing him.)What type of assessment is the behavior analyst likely conducting? A)An indirect assessmentB)A direct assessmentC)A functional assessment interviewD)A functional analysis

B. There are 3 components of a functional behavior assessment. They are 1) indirect assessments, which include interviews, checklists and surveys 2) direct assessments, which include physically observing the behavior without manipulating the environment and 3) functional analyses, which involves systematically manipulating the environment to see what effects it has on the behavior. In this example, the behavior analyst conducted a direct assessment since she observed the behavior without manipulating any environmental conditions to evoke the behavior.

Which explanation of behavior would a behavior analyst most likely use to describe why a student is physically aggressive towards his teacher? A)"He hits his teacher because he likes getting out of math assignments."B)"He hits his teacher because in the past it has resulted in him not having to complete his math assignment."C)"He hits his teacher because he is diagnosed with dyslexia which makes math more difficult for him compared to his typically-developing peers."D)"He hits his teacher because he knows he will get out of having to do his math assignment."

B. This is the least mentalistic of all the answer choices. "He knows" and "He likes" are mentalistic terms. Blaming it on an inner-identity or diagnosis involves circular reasoning and is mentalistic.

Matt, a BCBA®, was scrolling online and found a post from one of his high school friend's saying how they were having trouble with their son staying in the bed at night and causing problem behaviors during their nighttime routine. What should Matt do in this scenario? A)Comment on the post with suggestions for his friend.B)Ignore the post and move on.C)Give the friend a call and tell him to get the child diagnosed. D)Comment on the post with a link to an ABA clinic.

B. This one can be tricky, and as a friend, you may want to comment and provide advice so that your friend no longer is experiencing this problem. But, according to the BACB® ethics code, behavior analysts do not provide specific advice related to a client's needs in public forums.

A factory manager is measuring how quickly his employees can assemble a box of crayons. Julie assembles a box of crayons in 1 minute. Ronald takes 45 seconds to assemble a box of crayons. It takes Bonny 100 seconds to assemble a box of crayons. Joy can assemble a box of crayons in half the time that it takes Bonny. Who assembles boxes of crayons the fastest? A)JulieB)RonaldC)BonnyD)Joy

B. This one is pretty straightforward. Simply calculate the amount of time it takes each person to assemble a box of crayons, while putting every duration into seconds so you can easily see how long it takes each person. Julie=60 seconds. Ronald=45 seconds. Bonny=100 seconds. Joy=50 seconds.

For which behavior(s) would time-sampling recording procedures likely be used, rather than continuous recording procedures? A)The number of miles a person jogs in one hourB)TantrumsC)The number of times a person brushes his teeth in one dayD)(All of the above)

B. Time sampling is used when continuous measures (duration, event recording, rate, latency, etc.) would be too cumbersome or inaccurate. Time sampling is beneficial for behaviors 1) that do not have a clear stop/starting point and 2) occur too quickly to accurately track with continuous recording measures. Tantrums do not have a clear start and stop time; a tantrum may seem to be "over" but then continues on. Therefore, time sampling would be beneficial to use for tantrums. Amount of miles a person jogs in 1 hour is an example of rate. The number of times someone brushes their teeth in a day would likely be measured with a frequency count.

You have just begun working with a new client, named Riley, who displays multiple behaviors of concern. Of the following behaviors, which would you want to address first? A)Teasing his sister in the car, which has become very annoying to Riley's mother.B)Cutting his wrists with a paperclip, which has resulted in serious injury in the past, but now only occurs once per month.C)Slapping himself in the face, in a way that is relatively harmless but happens several times per week.D)Vocal stereotypy that results in "weird looks" from his peers.

B. When prioritizing problem behaviors and determining the order they will need to be addressed, the first and most important thing to consider is the dangerousness that the problem behavior is currently presenting. Since wrist cutting has resulted in serious injury in the past and it is still occurring once per month, Riley is in imminent danger of causing serious injury to himself. Even though slapping himself happens more frequently than wrist cutting, this is not causing significant injury at this time. Teasing his sister ranks near the bottom of these target behaviors as it is not dangerous. Finally, vocal stereotypy that results in "weird looks" from peers is not a danger to the client. Therefore, even though all of these behaviors should be analyzed and treated, the highest behavior priority at this time would be wrist cutting.

John Doe is a BCBA®. He is also a Board Certified Music Therapist (MT-BC®), a Certified Chiropractic Rehabilitation Doctor (CCRD®), and a Certified Ghost Hunter (CGH®). He also has Master of Arts degree (M.A.). How should John write his name on his business cards for the services he provides as a behavior analyst? A) John Doe, M.A., BCBA, MT-BC, CCRD, & CGH B) John Doe, M.A., BCBA C) John Doe, Master of Arts, Board Certified Behavior Analyst, Board Certified Music Therapist, Certified Chiropractic Rehabilitation Doctor, & Certified Ghost Hunter D) John Doe, M.A., BCBA, MT-BC, & CCRD

B. When working as a behavior analyst, it is important to ONLY use the credentials that represent behavior analysis. No matter how impressive the other credentials John received in his lifetime may be (especially Certified Ghost Hunter), they do not apply to the work he does as a behavior analyst. A BCBA should not advertise the other services they provide in their free time if they are not behavior-analytic in nature. John Doe, M.A., BCBA are the correct credentials to put on his business card when providing services as a behavior analyst.

Whole-interval recording tends to _________ the duration of a behavior. A) overestimate Correct answer B) underestimate C) accurately measure D) (None of the above)

B. Whole-interval recording typically underestimates the duration of a target behavior. Whole-interval recording is conducted by marking whether or not a target behavior occurred throughout an entire interval. If there is any amount of time where the target is not occurring during the interval, the interval is marked "-". Consider this: You are tracking a behavior using 10 minute intervals. The behavior occurs for the first 9 minutes of the interval before the learner stops engaging in the behavior. Using whole-interval recording, it will look like the behavior never occurred. However, it actually occurred for 9 minutes.

Whole-interval recording tends to _________ the rate of a behavior. A)overestimateB)underestimateC)accurately measureD)(None of the above)

B. Whole-interval recording typically underestimates the rate of a behavior. Whole-interval recording is conducted by marking whether or not a target behavior occurred throughout an entire interval. Consider this: you are monitoring how many times your client engages in a conversation with a peer at school. You monitor the behavior for an hour. The hour is broken up into 10 intervals. If he engages in a conversation with his peers for the entire interval, the interval would be marked successful. However, if he was not engaging in a conversation at any point in the interval, the interval would be marked "-". If your client initiated 8 separate conversations within an interval, the interval would still be marked "-" because the behavior of engaging in a conversation was not continuous. Therefore, whole-interval recording will likely underestimate the rate of a target behavior. Time sampling procedures, such as whole-interval recording, are not necessarily accurate. Accuracy refers to the true value of a certain behavior. Time sampling is simply a momentary point in time where you monitor the behavior. (It isn't accurate because the behavior likely occurs when you are not monitoring as well.)

A behavior analyst wants to use an intervention to address several of her client's behaviors. Which experimental design(s) would be appropriate for the behavior analyst to use? A)An alternating treatments designB)A changing criterion designC)A multiple baseline designD)(All of the above)

C. A multiple baseline across behaviors design would be appropriate to address several behaviors for a single client. Alternating treatments designs are used to assess multiple interventions on a single target behavior. A changing criterion design is used for a single target behavior.

A behavior analyst wants to determine the amount of money that would be most effective to use as a reward for a DRO procedure that is designed to decrease a client's physically aggressive behavior. To find out how much money to use as a reward for the DRO procedure, the behavior analyst should conduct a(n): A)experiment using a multiple baseline design.B)component analysis.C)parametric analysis.D)reinforcer preference assessment.

C. A parametric analysis analyzes the effectiveness of varying values of a particular intervention (varying levels of the independent variable). Therefore, analyzing how money to use as a reward for a DRO procedure is considered a parametric analysis. "Component Analysis" is incorrect since this involves seeing which components of an intervention package are crucial for the behavior change. "A reinforcer preference assessment" is incorrect since the reward of a DRO procedure is not a reinforcer by definition, since it is not delivered contingent on a target behavior. "Experiment using a multiple baseline design" is incorrect since multiple baseline designs involve one intervention being applied to multiple people, multiple behaviors, or multiple settings.

Written consent from a guardian should be obtained: A) ONLY before a more restrictive intervention is used. B) after a new intervention is used. C) before changing an intervention goal. D) (All of the above)

C. Be careful here, it is best practice to gain consent BEFORE any significant change to services provided is made. "After a new intervention is used" is incorrect since consent needs to be gained beforehand. "ONLY before a more restrictive intervention is used" is incorrect since "ONLY" indicates this is the only reason to gain consent from a guardian. (There are many other scenarios where gaining guardian consent is necessary. This includes any time a significant change is made to the behavior plans or if anything new is introduced to the current plan).

Athénaïs has completed her research project. She was assisted by Brittney, a student, whose contribution was relatively minor and would not be considered an author. How should she acknowledge Brittney's work? A)She doesn't need to as the contribution was minorB)Because Brittney contributed, she should be an authorC)Brittney should be appropriately acknowledged, such as, in a footnote or introductory statement.D)Each journal has different requirements. Athénaïs should follow the guidelines of each publication she submits her manuscript.

C. Behavior analysts acknowledge the contributions of others to research by including them as co-authors or footnoting their contributions. Principal authorship and other publication credits accurately reflect the relative scientific or professional contributions of the individuals involved, regardless of their relative status. Minor contributions to the research or to the writing for publications are appropriately acknowledged, such as, in a footnote or introductory statement.

Elian is studying for his BCBA® exam. A BCBA® in his workplace gave him a link to a blog that has real exam questions posted by recent test takers. What should Elian do in this scenario? A) Refuse to look at the blog. B) Use the blog as an exam preparation exercise. C) Report the site to the BACB® and refrain from reading any more of the questions. D) After taking the exam, report the blog to the BACB® if the questions are the same as the real BCBA® exam.

C. Behavior analysts adhere to all rules of the BACB®, including the rules and procedures required by BACB® approved testing centers and examination administrators and proctors. Behavior analysts must immediately report suspected cheaters and any other irregularities relating to the BACB® examination administrations to the BACB®. Examination irregularities include, but are not limited to, unauthorized access to BACB® examinations or answer sheets, copying answers, permitting another to copy answers, disrupting the conduct of an examination, falsifying information, education or credentials, and providing and/or receiving unauthorized or illegal advice about or access to BACB® examination content before, during, or following the examination. This prohibition includes, but is not limited to, use of or participation in any "exam dump" preparation site or blog that provides unauthorized access to real BACB® examination questions. If, at any time, it is discovered that an applicant or certificant has participated in or utilized an exam dump organization, immediate action may be taken to withdraw eligibility, cancel examination scores, or otherwise revoke certification gained through use of inappropriately obtained examination content.

Ozzy, a BCBA®, has a new supervisee, named Jenna. Ozzy wants Jenna to perform an assessment on a client. However, she has not performed this assessment before and has requested 1:1 support. Ozzy does not have authorization to provide that many hours of in-person oversight in a month.What is the best course of action Ozzy could take in this scenario? A) Ozzy should send Jenna articles and videos so that she can learn how to do the assessment, and ensure that she feels supported. B) Ozzy should remove Jenna from the case and replace her with an experienced supervisee who can complete the assessment. C) Ozzy is responsible for building assessment competency for Jenna; he should look for ways to train her to competently complete the assessment prior to asking her to complete the assessment by herself. D) Ozzy should instruct Jenna to do her best. He should take video for feedback and training and evaluate the results based on knowledge of the client.

C. Behavior analysts delegate to their supervisees only those responsibilities that such persons can reasonably be expected to perform competently, ethically, and safely. If the supervisee does not have the competency and experience to perform these duties, it is the BCBA's® responsibility to train the staff prior to them performing the task. In this scenario, replacing Jenna with an experienced supervisee may be necessary. But the simplest and first course of action would be training Jenna how to conduct the assessment. Providing articles and videos to Jenna would not be sufficient, since Ozzy would not know if Jenna can competently conduct the assessment. Finally, providing feedback based on video taken during the assessment is insufficient since Jenna should be trained PRIOR to conducting the assessment.

Shakeel's daughter has shared that there is a classmate who engages in "odd behaviors." Shakeel overhears that this student in his daughter's class is on the verge of being expelled while picking his daughter up from school. As a BCBA®, what should Shakeel do? A)The student may be in need of behavior analytic services. Shakeel should inquire with the teacher or principal if support is needed.B)Shakeel should ask his daughter to invite the student to their home so he can do an informal assessment and see what services may be appropriate.C)It is not appropriate to get involved.D)Shakeel should demand that the classmate get a diagnosis.

C. Behavior analysts do not engage, directly or through agents, in uninvited in-person solicitation of business from actual or potential users of services who, because of their particular circumstances, are vulnerable to undue influence. Organizational behavior management or performance management services may be marketed to corporate entities regardless of their projected financial position.

You are on an overnight flight. A child sitting directly in front of you will not stop crying. People sitting nearby are growing increasingly frustrated and the parents have tried everything to get their child to stop crying. You implement a DRO procedure where the child can watch videos on your cell phone for 1 minute if he can go 5 minutes without crying. He does not cry the rest of the flight. His parents call you a hero and ask for your contact information. What is wrong with this scenario? A)You should have attempted a less restrictive intervention first.B)You should have taught the parents how to do the intervention so they could do it by themselves in the future.C)You should have gained consent from the parents prior to providing any behavior-analytic services.D)(Nothing was wrong with this scenario)

C. Behavior analysts do not offer services to people under undue influence. They must gain consent to conduct any interventions or assessments. Prior to providing services, a behavior analyst should also talk with the parents about the fees of services, the terms of services, behavioral goals, etc.

A behavior analyst notices that a child she is working with has a red rash on his arm. The child's mother asks the behavior analyst what she thinks is causing the rash. Choose the most appropriate response that the behavior analyst should make. A) "That's poison ivy. My kids get it every year. Just rub some anti-itch lotion on it and it'll clear up in a couple days." B) "Looks like poison ivy to me. I'll modify his behavior plan to ensure he doesn't itch his arm, since itching will cause it to spread." C) "It looks like poison ivy to me but that is not my area of expertise. I suggest taking him to his physician to get it checked out." D) "Looks like poison ivy to me. Let's continue to track his behavior for a couple of days and see how it affects behavior. If his behavior changes, he may need to see a physician."

C. Behavior analysts do not provide advice in areas that are not behavior-analytic in nature (this is a medical issue). The mother should take him to see his physician since that person is qualified to provide an opinion about treatment. A behavior analyst would not delay seeking medical treatment by collecting data for a couple more days. They should not offer advice on which lotion to use. Also, they should not add the target behavior of scratching to the behavior plan since it does not get at the root cause (there would be no reason to address the scratching if the doctor gave him medicine to clear up the rash).

Bongani accidentally submitted an RBT® competency assessment too early having not reviewed case notes with the RBT® but accidentally checking that it was complete. What should Bongani do? A) Work with the RBT®, ensure that they are competent in taking case notes. B) Redo the entire assessment. C) Inform the BACB®. Complete the competency assessment in its entirety and resubmit. D) Inform the BACB® of the reporting error.

C. Behavior analysts ensure that inaccurate information submitted to the BACB® is immediately corrected.

Ruby is doing a research assignment. She is very cognizant not to plagiarize. What must she do in order to ensure she doesn't plagiarize? A)Ruby must avoid using direct quotesB)Ruby must avoid paraphrasingC)Ruby must fully cite the work of others where appropriate.D)All of the above

C. Behavior analysts fully cite the work of others where appropriate.

Hussain is an OBM BCBA®. His agency requires him to complete an assessment for a client he is taking over from another BCBA® who had to leave the agency suddenly. The agency has given Hussain the file which has a lot of data, the previous assessment, and a progress report. What must he consider before completing the assessment? A)Even with a complete file, Hussain should consult with the previous BCBA® to inform his written assessment.B)Nothing in that report is valid as it is all part of the previous assessment cycle. Hussain will need to start from the very beginning and conduct an intake.C)Hussain needs to involve the client in the planning of and consent for this upcoming assessment.D)If the data is complete; he can use what he has been given to write a report as assessments are data-based.

C. Behavior analysts involve the client in the planning of and consent for behavior-change programs.

Dominique is writing a supervisee contract for a supervisee at her agency. What must she include? A)Dominique doesn't not need write a supervisee contract. If she follows The Behavior Analyst Certification Board's (BACB's) Professional and Ethical Compliance Code for Behavior Analyst's Section 4 (Responsibility to Supervisees and Trainees), she will be doing what is necessary to be a responsible supervisor. B)Dominique would follow agency guidelines as to what expectations are necessary for supervisors.C)Dominique should have a contract that has clear written description of the purpose, requirements, evaluation criteria, conditions, and terms of supervision.D)Dominique should go over her and the supervisees responsibilities on the outset of supervision (first meeting) of their responsibilities on the onset of supervision. The conversation only needs to be documented.

C. Behavior analysts provide a clear written description of the purpose, requirements, evaluation criteria, conditions, and terms of supervision prior to the onset of the supervision.

Serena is a BCBA® and is unable to come to an agreement with an insurance company on what constitutes an appropriate level of service for a client. What should she do in this situation? A) The client's parents and the agency are content with what the insurance is recommending. Comply with the insurance company's recommendation. B) If an agreement cannot be reached, Serena is ethically obligated to leave the case rather than implementing a level of service that is inappropriate based on the data she collected. C) Assess what is in the best interest of the client. Serena needs to consider harm from abandonment and moving forward with what she has assessed as a poor level of service delivery hours and make her best ethical decision. D) Serena should request a second opinion from a respected colleague.

C. Behavior analysts put the client's care above all others and, should the third party make requirements for services that are contraindicated by the behavior analyst's recommendations, behavior analysts are obligated to resolve such conflicts in the best interest of the client. If said conflict cannot be resolved, the behavior analyst's services may be discontinued following appropriate transition.

A mother is tracking how often her daughter plays the piano. After breakfast, her daughter plays the piano for 30 minutes. 2 hours later, her daughter begins playing again. This time, she plays for 1 hour. What is the interresponse time of her daughter playing the piano? A)180 minutesB)150 minutesC)120 minutesD)90 minutes

C. For interresponse time, we are looking at the time between her ending one playing session and her starting to play the piano again. This example says that she stopped playing the piano for 2 hours before she started playing it again. (The duration of time she spent playing the piano is irrelevant.) 2 hours=120 minutes.

If the functional analysis data reveals that the behavior occurs at high rates in all conditions, or does not occur at all in any of the conditions, then: A)the function of the behavior is automatic reinforcement.B)an intervention is not necessary.C)the functional analysis was inconclusive.D)the function of the behavior is attention.

C. If a behavior occurs at high rates in all of the conditions, the function is either automatic reinforcement OR the function could not be determined. Since this question stated that it occurs at high rates in all conditions or not at all in all conditions, "automatic reinforcement" is incorrect. If the behavior never occurred at all during the functional analysis, we could not determine that it is automatically reinforced. If the function of the behavior was attention, it would likely occur most frequently in the contingent attention condition. An intervention may still be necessary, especially if the target behavior is occurring at high rates in all conditions.

You are using a partial-interval recording procedure to track a client's verbal aggression. During a 5-minute interval, he was verbally aggressive 7 separate times. What would you record on the tracking sheet for that interval? A)The number "7"B)7 talliesC)Whatever symbol that tracking form uses to denote that the behavior occurredD)Whatever symbol that tracking form uses to denote the behavior did not occur throughout the entire interval

C. In partial interval recording, we are simply marking whether behavior did, or did not occur during that interval. Different symbols are used to denote that the behavior occurred during that interval depending on the specific tracking form you are using. "7" and "7 tallies" is incorrect since you are just marking whether the behavior occurred or not, and not the number of times it occurred. "Whatever symbol that tracking form uses to denote the behavior did not occur throughout the entire interval" is incorrect since this would be whole-interval recording and not partial-interval recording.

Every time Hank goes to pull at the blinds in the kitchen his mother shouts, "no!" In the future, Hanks behavior of pulling at the blinds decreases. What procedure did Hank's mother use to decrease the behavior of pulling the blinds? A)Negative punishment.B)Negative reinforcement.C)Positive punishment.D)Positive reinforcement.

C. In positive punishment, an aversive stimulus is presented, contingent on a behavior, that results in a decreased frequency of that behavior in the future. In this example, Hank's mother said 'no!' (presenting an aversive stimulus) contingent on the target behavior.

Helga works with a client whose handwriting is illegible. Helga conducts an experiment to determine whether a new mobile app can help increase her client's handwriting skills. Her study shows a clear functional relation between the independent variable (the mobile app) and the dependent variable (her client's handwriting skills). A)DescriptionB)PredictionC)ControlD)(None of the above)

C. In this scenario, Helga is able to demonstrate that there is a clear functional relationship between the independent variable and the dependent variable. Therefore, Helga is able to control the environment (use the mobile app vs. not use the mobile app) and this directly correlates to her client's handwriting skills. Helga's experiment shows that not only is the increase in handwriting skills correlated with the mobile app; it also shows that the mobile app is the cause of the increases in handwriting skills

Whenever Osiris is hungry, he hands a laminated picture of food to his mother and she gets him food. What would be an example of response generalization? A)Osiris hands the laminated picture to his dad when he is hungry.B)Osiris hands the laminated picture to his babysitter when he is hungry.C)Osiris gets food for himself when he is hungry.D)(All of the above)

C. Response generalization occurs when a learner engages in an untaught response to access a reinforcer that the taught response also gains. In this example, Osiris hands a laminated picture of food to his mother when he is hungry and this results in a reinforcer (food). Response generalization would be any new response that results in him gaining food. Handing the laminated picture of food to another person is not response generalization, it is stimulus generalization. Getting food for himself is considered response generalization since it is a novel response that resulted in gaining access to food.

A behavior analyst talks with his client's parents about how often their child engages in the targeted response, how long the target response occurs, what they do when the target response occurs, and what the target response looks like. What type of assessment procedure is the behavior analyst most likely using? A)A behavioral rating scaleB)ABC data recordingC)An indirect assessment procedureD)A direct assessment procedure

C. In this scenario, it sounds like the behavior analyst is conducting a structured behavioral interview. Interviewing the parents about the function and other details of the target behavior is considered an indirect assessment procedure since the behavior analyst is not actually observing the target behavior as it occurs. "A direct assessment procedure" is incorrect because it would involve directly observing the behavior, rather than simply asking questions about it. "ABC data recording" is incorrect because this involves directly observing and recording the behavior as it occurs, which is not what happened in this scenario. "A behavioral rating scale" is incorrect with the information given; this scenario never mentioned rating scales. This scenario sounds more like a behavioral interview regarding the function of the target behavior, therefore an indirect assessment procedure is the best answer choice.

Brianna is a BCBA® who is working with a client who is also receiving pseudoscientific intervention through a Facilitated Communication (FC) facilitator. The parent has shared that her child, during an FC session, "wrote" that he prefers FC sessions to his ABA sessions. What should Brianna do? A) Provide the parent an ultimatum. ABA and FC cannot coexist and she has to make a choice as to what therapy she wants for her child. B) Provide literature on the perils of FC. C) Discuss the goals and data and demonstrate the effectiveness of ABA and generalization of skills across environments. Acknowledge the reinforcement contingencies and discuss effective service delivery from the parent's prospective. D) Learn about FC, attend sessions, make an effort to be a facilitator yourself, and practice philosophical doubt. Depending on the results, FC may be effective for this specific client or, if not, you can discuss, from experience, why ABA is superior to FC.

C. In those instances where more than one scientifically supported treatment has been established, additional factors may be considered in selecting interventions, including, but not limited to, efficiency and cost-effectiveness, risks and side-effects of the interventions, client preference, and practitioner experience and training.

Three tests that must be met before __________ can be considered valid include: the capacity to decide, the decision must be voluntary, and the person must have adequate knowledge of the aspects of the treatment. A)ethical issuesB)uninformed consentC)informed consentD)interobserver agreement

C. Informed consent is considered valid if the person has the capacity to decide, it is a voluntary decision, and there is knowledge of the treatment.

An experiment that demonstrates high __________ shows convincingly that the changes in the behavior are a result of the implementation of the independent variable (and not some other confounding variable). A)external validityB)correlationC)internal validityD)prediction

C. Internal validity refers to the degree to which an experiment has demonstrated a functional relation between the independent and dependent variable (meaning, we can say convincingly that the change in the behavior was due to the independent variable, rather than due to some other cause).

Josie is addicted to smoking cigarettes. Her boyfriend hates that she smokes. He chastises her whenever he sees her smoking. Therefore, she stopped smoking cigarettes at home and now smokes more cigarettes at work. This is an example of: A)Stimulus ControlB)Behavioral ContrastC)(Both A and B)D)(Neither A nor B)

C. Josie does not smoke at home; she only smokes at work. This demonstrates stimulus control (the presence of work is an Sd and the presence of her boyfriend is an Sdp or S-delta). She now only smokes at work, therefore stimulus control is present. This could also be an example of behavioral contrast. Her behavior of smoking is on an FR1 schedule of reinforcement at work because every time she smokes she is reinforced. Since she is chastised for smoking at home, it occurs less frequently there. Since the behavior of smoking decreases in rate at home, it therefore increases at work. This is an example of behavioral contrast and it also demonstrates a degree of stimulus control.

Which of the following is considered a behavior, according to methodological behaviorists? A)Pain from a headacheB)Sore muscles after a workoutC)Lifting weightsD)Thinking about which activity you will perform during a workout

C. Methodological behaviorists do not consider thoughts and feelings to be behavior, therefore, "lifting weights" is the correct answer because it is the only observable behavior (the other options are private events).

A teacher is measuring whether a student stays in his seat during teaching lessons as instructed. Every 5 minutes, she records whether the student is in his seat or not at that time. Which measurement procedures is she using? A)Event recordingB)Partial-interval recordingC)Momentary time samplingD)Whole-interval recording

C. Momentary time sampling is correct since she is simply observing briefly at scheduled intervals and recording whether or not he is sitting down in his seat. Whole-interval recording would involve watching him continuously to see if he stays in his seat. Event recording would involve simply marking down each time he sat in his seat. Partial interval recording would involve marking whether he sat in his seat at any point during the interval; since she only observed briefly every 5 minutes, it is not partial interval recording.

In an alternating treatments experimental design, what could a behavior analyst do to minimize the risk of multiple treatment interference? A)Return to baseline for several days before switching to another experimental design.B)Return to baseline for several weeks before switching to another experimental design.C)Make all experimental conditions highly discriminable from one another.D)Make all experimental conditions indiscriminable from one another.

C. Multiple treatment interference may occur when experimental conditions are so similar that the learner cannot differentiate between experimental conditions. To minimize the risk of this, an experimenter should make each experimental condition very different from the others to help the learner discriminate between experimental conditions. An alternating treatments design requires that experimental conditions be rapidly changed from one condition to another. Therefore, baselines of several days or weeks would not be recommended.

When using a changing criterion experimental design, what can you do to strengthen the demonstration of experimental control? A)Use the intervention with several behaviors that are significantly different than one another.B)Use the intervention with several behaviors that are similar to one another.C)Reverse the direction of the criterion changes.D)Wait for responding to stabilize with one target behavior prior to applying the intervention to the next target behavior.

C. Reversing the direction of the criterion change will further demonstrate experimental control, if the behavior changes direction to match that of the criterion change. Changing criterion designs involve one behavior and one intervention (but the criteria of the intervention changes). Therefore, all of the other answer choices are incorrect.

When using an intervention to teach a child how to ride a bike, why shouldn't a behavior analyst use a reversal experimental design? A)ABA should not be used to teach bike riding.B)Teaching a client how to ride a bike and then reversing the intervention could result in negative side effects, such as his peers making fun of him.C)Reversing an intervention that is used to teach bike riding would not result in a deterioration of the skill during the baseline phases and therefore would not demonstrate experimental control.D)Teaching a child how to ride a bike in reverse could result in injury since the child cannot see where he/she is going.

C. Since riding a bike is a skill that you cannot necessarily "unlearn," the client's bike riding would not diminish in the baseline phases. It would therefore not demonstrate experimental control. Since a reversal design in this case would not result in a reduction in bike riding skills, this experimental design would likely not result in peers picking on him for not being able to ride a bike. "Teaching a child how to ride a bike in reverse could result in injury since the child cannot see where he/she is going" is incorrect since a reversal experimental design does not mean that you teach the skill in reverse. Finally, ABA could potentially be used to teach someone how to ride a bike.

A manager of a doll factory sits in his office most of the day. However, a few times per day he will randomly walk out onto the factory floor and provide praise to each worker who assembles a doll. What schedule of reinforcement is the manager using? A)Variable ratioB)Fixed ratioC)Variable intervalD)Fixed interval

C. Since the manager goes out onto the floor of the factory randomly throughout the day, and reinforces the first correct response at that time, it is a variable interval schedule or reinforcement. It is not a fixed ratio schedule because although the first response was reinforced and it appears to be an FR1 schedule, it actually is not since previous responses in the absence of the manager were not reinforced. "Fixed interval" is incorrect since the manager goes out on the factory floor at random times. "Variable Ratio" is incorrect since the reinforcement is on a ratio schedule.

A behavior analyst is conducting a functional analysis. When the target behavior occurs, she ignores the behavior. Which condition of a functional analysis could the behavior analyst be conducting? A)The play conditionB)The alone conditionC)(Both A and B)D)(Neither A nor B)

C. The alone condition results in ignoring or neutrally redirecting the behavior when it occurs. The play condition also results in ignoring or neutrally redirecting the behavior when it occurs.

A behavior analyst is using a whole-interval recording method to see if her client will stay in his seat the entire class. Each interval is 5 minutes long and there are 12 total intervals.Under which scenario would the behavior analyst be required to monitor her client's behavior for the entire hour-long class? A)If he sat down in his seat at any point during the interval.B)If he did not show up to school this day.C)If he stayed in his seat for the entire hour.D)If he used the bathroom (or otherwise left his seat) at least one time per interval.

C. The behavior analyst is monitoring whether or not her client stays in his seat. Since she is using a whole-interval recording method, she is monitoring if he stayed in his seat the ENTIRE interval. In this question, we are looking for a reason why the behavior analyst would have to monitor her client's behavior for the entire hour. In whole-interval recording, the behavior analyst only has to monitor the behavior if it occurs throughout the entire interval. If he stood up at any point during the interval, the behavior analyst would no longer have to monitor his behavior since that interval would be marked unsuccessful. If he was not sitting in his seat at the beginning of the interval, that interval would also be marked unsuccessful. The only reason she would have to monitor his behavior for the entire hour (using whole-interval recording) would be if he was in his seat for the entire hour.

This dimension of applied behavior analysis (Baer, Wolf, & Risley, 1968) means that the behavior changes that are achieved with the intervention should be long-lasting and should generalize to other settings: A)Generalization.B)Discrimination.C)Generality.D)Applied.

C. The generality dimension of applied behavior analysis means that the behavior changes that are achieved with the intervention should be long-lasting and should generalize to other settings. "Generalization" sounds like generality, but the terms are different. "Discrimination" and "Applied" are also incorrect.

All of the following are levels of scientific study, except: A)DescriptionB)PredictionC)Radical BehaviorismD)Control

C. The three levels of scientific study are: description, prediction, and control. Radical behaviorism is an approach to psychology that address all psychology in terms of the principles of behavior (including thoughts and feelings).

Which experimental design(s) are variants of a multiple-baseline design? A)A multiple probe designB)A delayed multiple baseline designC)(Both A and B)D)(Neither A nor B)

C. These are both variations of the multiple baseline design.

You are observing a client at his physical therapy session. The physical therapist's assistant called in sick today. The physical therapist is having a difficult time without her assistant there. She asks you to hold your client's leg down while she stretches out his ankle. How should you respond in this scenario? A) Hold your client's leg down B) Hold your client's leg down if your client says that it is okay C) Politely decline D) Contact the BACB®

C. This is a tricky situation. But we believe the best option in this scenario is to politely decline to help the physical therapist. Behavior analysts must stay within their area of competence. Since you have never been trained on these physical therapy procedures, it is likely unethical to play the role of the physical therapist's assistant. It could result in your client being injured because you are not trained on the procedures. Even if your client consents, it does not mean you should necessarily do it. (Ex: Your client may consent for you to prescribe him medications, but you still cannot do so.) You likely wouldn't have to contact the BACB® in this situation since they do not oversee the services rendered by physical therapists.

Colby is taught to say "bike" when she sees a picture of a bike. Then, without training, she points to the picture of the bike when she hears her therapist say "bike".Colby pointing to the bike, upon hearing the spoken word "bike", is an example of ___________. A)reflexivityB)transitivityC)a derived stimulus relationD)matching law

C. This is an example of a derived stimulus relation because Colby learned the relationship between two stimuli without being directly taught that the relationship exists. Colby was taught that seeing a picture of a bike and saying "bike" are related. Then, without further training, she inferred that hearing the word "bike" and a picture of a bike are related. Therefore, this is a derived stimulus relation. More specifically, it is an example of symmetry. "Reflexivity" is incorrect since this would involve the same stimulus-stimulus pairing to occur (ex: A=B). "Transitivity" is incorrect since this would involve an untrained relation deriving from two other stimulus-stimulus relations (ex: A=B. And B=C. Therefore A=C). "Matching law" is incorrect since this involves responding in a way that matches the level of reinforcement that is available in two separate schedules of reinforcement, which does not apply to this scenario.

A BCBA®, who just passed the certification exam, is assigned her first client. The client is a child with autism who engages in self-injurious behavior. The BCBA® has never worked with a child who is self-injurious. However, the BCBA's® brother engages in self-injurious behavior and she has helped him in the past. Does this BCBA® have the relevant experience to work with this client? A)Yes, she should take on this client. She has the relevant training and experience to work with this population. B)Yes, she likely has the relevant education and experience to take on this client. If she ever feels unprepared, she should reach out to a BCBA® who has more relevant experience in this area.C)No, she should not take on this client. Just because her brother was self-injurious does not mean that she has experience using behavior-analytic procedures to shape the behavior of children with autism who are self-injurious.D)No, she must be supervised by a BCBA at all times for 1 year until she can ensure her competence in this area.

C. This is the best answer choice for this scenario. The behavior analyst should not take on this client if she does not have the relevant training and experience using behavior analysis to shape the behavior of children with autism who are self-injurious. So, what could she do in this situation? She could politely decline the referral or she could ask for a BCBA® who is competent in this area to supervise her. A BCBA® does not need to be supervised by another BCBA® for an entire year. When she demonstrates competency in the area, the supervising BCBA® could fade his/her supervisory role.

Vick loves basketball. He wants to improve his free throw percentage so that he can make his high school's basketball team. On the first day, Vick took 10 free throw shots and made 60% of these shots. On the second day, Vick took 5 free throw shots and made 2 of them. On the final day, Vick took 10 free throw shots and made 80% of these shots. What is Vick's overall free throw percentage from these 3 days? A)44%B)53%C)64%D)(None of the above)

C. To find the correct answer, we must determine the total amount of shots Vick took AND the total number of shots he made in the basket. He shot 25 total shots and made 16 of them. 16/25=0.64x100=64%

When you begin working with a new client, it is necessary to tell them: A)what college you attended.B)your GPA in graduate school.C)how much services cost and what methods of payment you accept.D)(All of the above)

C. hen you first meet the client, you should provide them with contact information, discuss cost of services, how to discontinue services, how to file a complaint, etc. It is not always necessary to disclose what university you attended or what grades you received during college. If this information is requested by a client, it may be revealed if you believe it is necessary and appropriate.

Sheera is a BCBA® with a full caseload. She has been asked to temporarily take on the caseload of a colleague who has recently been fired. What should Sheera do in this scenario? A)She should do her best to oversee the clients of her colleague until they hire someone new to take over those cases.B)She should assess which clients could have their oversight on hold to ensure the number of clients she is supervising remains consistent (prioritize those cases that are needing more supervisory support). C)She should make a case that taking on extra cases will have a negative effect on her current caseload which should be avoided.D)She should asks colleagues to share in the extra caseloads so that she is not doing all the clients of the recently departed BCBA®.

C.Behavior analysts take on only a volume of supervisory activity that is commensurate with their ability to be effective.It should be noted that having too many cases will limit your effectiveness as a BCBA®. If you are spread too thin, you are not able to uphold your responsibility to the behavior change program. Asking for a lighter caseload is often challenging and may be uncomfortable but it is necessary for the betterment of your clients.

Which explanation of behavior would be most acceptable to behavior analysts? A)"Logan tells jokes because he is the class clown."B)"Logan tells jokes because he likes peer attention."C)"Logan tells jokes because he knows his classmates will laugh."D)"Logan tells jokes because he receives attention from his peers."

D. "Logan knows..." and "Logan likes..." are incorrect since they use a mentalistic explanation. "Because he is the class clown" is incorrect because it does not explain the function of the behavior and "class clown" is subjective.

Pablo notices that he gets the best responding out of his client Jack right before lunch time, when he is hungry. Jack almost always chooses an edible reinforcer from his choice board and Jack is able to complete all of his trials quickly to earn the reinforcer. What is the example of hunger in this scenario? A)Discriminative StimulusB)S-DeltaC)Antecedent PromptD)Motivating Operation

D. A motivating operation is any stimulus or condition that affects learning and performance with respect to a particular reinforcer. In this example, Jack's hunger was a motivating operation that temporarily causes an increase in his performance so that he can more quickly earn access to the reinforcer.

For children, bubbles are a: A)reinforcer.B)preferred item.C)punisher.D)stimulus.

D. A stimulus can be any event, condition, or item that an organism can contact through any of its 5 senses (sight, hearing, smell, taste, and touch). If it is something that you can see, hear, smell, taste, or touch/feel then it is a stimulus. We don't know if bubbles function as a reinforcer, preferred item, or punisher for a child without further testing, but we do know that bubbles are a stimulus.

Which of the following is an example of a stimulus class? A)shapes of tables (e.g., square, round, rectangular, etc.)B)types of flowers (e.g., daisies, roses, etc.)C)breeds of dogs (e.g., lab, poodle, etc.)D)all of the above

D. A stimulus class is a set of stimuli that share a common physical property (i.e., size, color, weight, etc.).

All service notes should be written in a manner such that: A) they may be used in court proceedings. B) they may be read by a behavior analyst who will someday implement these interventions. C) the behavior analyst can refer back to his notes and determine which interventions did or did not work. D) (All of the above)

D. All documentation should be written in a way that if subpoenaed, they would be able to be used in court. They should also be written so that you, or another behavior analyst, could refer back to these documents and determine what strategies did and did not work.

Rich was so excited when his client Joe finally was able to mand for a preferred item independently! Rich took out the camera at the clinic and recorded the progress that Joe was making to upload to the clinic website and social media account. Rich then texted Joe's mom to make sure she saw the video of Joe manding independently, and Joe's mom was so excited. What is wrong with this scenario? A)Rich should have obtained informed consent before posting.B)Rich should have included a disclaimer that informed consent was obtained and that the information should not be captured and reused without express permission.C)Rich should not have published on social media channels or posted in a manner that increases the potential for sharing.D)(All of the above).

D. All of the above choices are examples of what is incorrect in this scenario. According to the BACB® ethics code, when publishing information and/or digital content of clients on their professional social media accounts and websites, behavior analysts ensure that for each publication they: (1) obtain informed consent before publishing, (2) include a disclaimer that informed consent was obtained and that the information should not be captured and reused without express permission, (3) publish on social media channels in a manner that reduces the potential for sharing, and (4) make appropriate efforts to prevent and correct misuse of the shared information, documenting all actions taken and the eventual outcomes.

Leesa already has 10 clients on her caseload, as well as a behavior technician that she provides behavior analytic supervision to twice per week. One of her previous families reaches out to Leesa and asks her if she could come back to their case (the family reports that things really are not working well with the new behavior analyst). What should Leesa do in this scenario? A) Tell the family she can accept the client. B) Tell the family that things will get better with the new behavior analyst. C) Provide suggestions of what to do over the phone. D) Tell the family that are not available to accept the client at this time.

D. Behavior analysts only accept clients whose requested services are within their identified scope of competence and available resources (e.g., time and capacity for case supervision, staffing). Leesa is already busy with her current caseload and supervisee, and should not accept another client at this time. She should not talk poorly about the other behavior analyst or provide services over the phone.

When using a changing criterion experimental design, you can do all of the following to strengthen demonstration of experimental control, EXCEPT: A)vary the length of phases.B)vary the magnitude of the criterion changes.C)reverse the direction of the criterion changes.D)wait for responding to stabilize with one target behavior prior to applying the intervention to the next target behavior.

D. Changing criterion designs involve one behavior and one intervention (but the criteria of the intervention changes). Therefore, intervening on another behavior would not be done in a changing criterion design, so this answer choice is incorrect. The other options can all be used to increase the likelihood of demonstrating experimental control.

A behavior analyst is working with a client, named Vera. Vera has a history of faking seizures. She does this several times per week. A behavior analyst determines that she fakes seizures because she receives a lot of attention for doing so. Vera has no history of having real seizures.Today, the behavior analyst took Vera to a fast food restaurant as a reward. While eating, Vera suddenly fell out of her seat and began shaking on the ground. What should the behavior analyst have done in this situation? A) Keep eating and ignore Vera's behavior. B) Instruct everyone in the restaurant to remain calm and ignore her since she fakes seizures frequently. C) Perform CPR. D) See if Vera is okay and call 911 if necessary.

D. Even though Vera has no history of seizures, this behavior should not be ignored. Vera may be choking on her food, may be having adverse side effects to a new medication, may have blood sugar issues, etc. The behavior analyst should not perform CPR unless Vera is in cardiac arrest. Performing CPR before confirming there is a medical emergency can result in serious injury to Vera. The behavior analyst should subtly determine whether or not Vera is having a medical emergency and call 911 if necessary.

John is a behavior analyst who works with a client named Larry, who lives in a rural area. During a session with Larry, his guardian says to John, "Hey, I remember you saying you loved working with computers. Mine stopped working and no technicians will drive out here to fix it because we live in the middle of nowhere. Could you take a look at it to see what is wrong?" What would be the most appropriate response for John to give? A)"Sure, no problem. I can take a look and tell you what is wrong with it. But I cannot fix it since this would be considered a dual relationship."B)"I apologize. Since I already work as a behavior analyst for your family, I cannot ethically work on your computer. If you'd like, I can bring it into town with me and have a computer shop look at it and then I can bring it back to you when I come back for my next session."C)"Sure. But I cannot do this on my company's time. I can come over this weekend and take a look."D)"I apologize. But according to the ethical code that I have to follow, I cannot work on your computer since I already provide behavioral supports services for your family."

D. Even though this sounds a little bit rude or strict, this is the best response. A behavior analyst should avoid dual relationships at all costs because they could negatively impact the services you provide. This includes taking the computer to someone who can fix it, fixing it on your day off, or diagnosing what is wrong with it even if you are not the one who fixes it.

Alex does not like his job. He shows up to work every day, but will often fake sick by telling his boss he has just vomited in the restroom. The boss always sends him home immediately due to their company policies regarding illness. Which of the following data collection procedures would be most accurate for Alex's boss to use to record how often Alex is faking sick? A)Whole-interval recording with short intervalsB)Whole-interval recording with long intervalsC)Momentary time samplingD)Event recording

D. Event recording would be the most ACCURATE option to track how many times Alex fakes sick. The boss could simply make a tally each time Alex fakes sick. This would be the most accurate and easiest recording procedure. Whole-interval recording would involve recording whether or not he faked sick during an entire interval. The behavior likely would not occur for an extended duration because he fakes sick and goes home immediately. Therefore, whole-interval recording would not accurately track this faking sick behavior (no matter the duration of the intervals). Momentary time sampling would involve the boss going up to Alex at predetermined intervals to see if he was faking sick. This is likely counterproductive and more work than simple event recording.

When would a behavior analyst conduct a functional behavior assessment? A)When a client the behavior analyst has been working with for years displays a new problem behaviorB)When he receives a client from another behavior analyst who has previously conducted a functional behavior assessmentC)When he receives a new client who has never received behavioral supportsD)(All of the above)

D. FBAs are conducted to determine the function of a problem behavior. Therefore, they should be conducted when the behavior analyst begins working with them (even if an FBA was previously conducted by another behavior analyst, since the function of the behavior may change over time). Also, if a new problem behavior arises, the function of that behavior will need to be analyzed.

When conducting the contingent escape phase of a functional analysis, what type of demand should the behavior analyst use? A)A highly preferred demandB)A demand that has resulted in aggression in the pastC)A neutral demandD)A demand that has a history of evoking the problem behavior

D. When delivering demands in the contingent escape phase of a functional analysis, a behavior analyst should use a demand that has evoked the target behavior in the past. Since we are trying to evoke the target behavior in a functional analysis, demands that have a history of evoking the target behavior are the most beneficial to use. "A demand that has resulted in aggression in the past" is less correct. Since a functional analysis can be used to determine the function of many behaviors (not just aggression), this answer option is less correct.

Igor is taught that he can leave math class after the school bell rings. One day in the middle of class, his teacher's phone rings. Igor walks out of class. He no longer walks out of class when the school bell rings and only walks out of class when his teacher's phone rings.Igor walking out of class is an example of: A)overgeneralization. B)response generalization.C)response maintenance.D)faulty stimulus control.

D. Faulty stimulus control means that a behavior is constantly and solely evoked by an incorrect antecedent stimulus. Overgeneralization means that the behavior is under a stimulus class that is too broad. That is, the response is occurring under similar but incorrect stimuli as well as the correct stimulus. The correct discriminative stimulus in this example is the school bell. The school bell should evoke the response of Igor walking out of class. However, since he walked out of class when his teacher's phone rang, this shows that Igor's behavior was evoked by an incorrect stimulus. Since the behavior is evoked only by an improper stimulus, this is considered faulty stimulus control. "Response generalization" is incorrect since the same behavior (walking out of class) is being emitted. Response generalization involves emitting a new behavior that is functionally equivalent. "Response maintenance" is incorrect since this means a skill that was taught with an intervention continues to occur under the correct conditions after the intervention has been discontinued. This is not response maintenance because Igor is not engaging in the trained response.

You show up to your client's home and the client's parents seem eager to see you. They say, "Joey has not been eating for a day and a half. Every time we put food in front of him, he throws it and cries. Could you please fix this?" A)"Sure, I'll be glad to help. Let me first run a functional assessment to see why this behavior is occurring."B)"Sure, that's what I'm here for. Let's pair healthy foods with his preferred snacks so that he becomes motivated to eat again."C)"I can't help with food refusal."D)"Sure, I'd love to help. Because he hasn't eaten since yesterday, there might be something else going on such as an illness, an unfavorable side effect of a new medication, etc. Why don't you first take him to his physician to rule out any medical causes? Then I can come back and work on it if there is no medical explanation for his food refusal."

D. For some behaviors, a behavioral intervention is unnecessary. There may be something medical going on that is suppressing his appetite. A behavior analyst would not step out of his/her boundaries by providing advice before a medical cause is ruled out. If there was an illness or an unwanted side effect to a new medication, the doctor could "fix" this situation without a behavioral intervention. As behavior analysts, we must rule out medical causes to the intervention first by referring them to their doctor. You would do this before running a functional assessment or any particular intervention. Research shows that behavioral interventions have been successful for treating food refusal, but the strategies should not be used until a medical assessment has ruled out a medical cause for the food refusal.

For which of the following baseline(s) is it appropriate to start the intervention phase? A)A stable baselineB)A baseline that has an increasing trend, if the goal is to decrease that behaviorC)A baseline that has a decreasing trend, if the goal of the intervention is to increase that behaviorD)(All of the above)

D. Generally, you would want a baseline to be stable prior to starting the intervention phase. However, you can also demonstrate a functional relation if you start the intervention phase that is counter to the trend of the baseline. If a baseline was decreasing in trend and the intervention was designed to increase the behavior, then it is still appropriate to implement the intervention. If a baseline had an increasing trend and the intervention is designed to decrease the behavior, then it is also appropriate to start the intervention if necessary.

Zack is using a multiple baseline across behaviors design with his client. After all three behaviors show a steady baseline, he applies the intervention to the 1st behavior. The 1st behavior increases while the other behaviors remain stable. After steady responding is gained, he introduces the intervention to the 2nd behavior. The 2nd behavior increases to the level of the 1st behavior and the 3rd behavior still remains stable. Finally, the intervention is applied to the 3rd behavior. It appears that the intervention increased the rate of responding for all 3 behaviors. To demonstrate experimental control, Zack needs to show prediction, verification, and replication in his study. Which part of this study demonstrates verification? A)When the intervention was applied to the 2nd behavior, results were similar to the 1st behavior.B)When the intervention was applied to the 2nd behavior, the 1st behavior returned to baseline levels.C)When the intervention was applied to the 3rd behavior, results were similar to when the intervention was applied to the 1st and 2nd behavior. D)When the intervention was applied to the 1st behavior, the 2nd and 3rd behavior remained stable

D. In a multiple baseline across behaviors design, the intervention is applied to 1 behavior at a time. When all 3 behaviors have reached a stable baseline, the intervention was applied to the 1st behavior and the behavior increased. Because behavior is relatively constant if no intervention is applied, we can predict that the 1st behavior would have remained stable if the intervention was never introduced. This prediction is verified by the 2nd and 3rd behavior remaining unchanged when the intervention was applied only to the 1st behavior. (Because the other behaviors remained unchanged, the prediction that the 1st behavior would have remained unchanged if the intervention was not implemented was verified.) Replication was demonstrated when similar effects were achieved after the intervention was applied to each behavior. "When the intervention was applied to the 2nd behavior, the 1st behavior returned to baseline levels" is incorrect since this was not mentioned in the example and this is not how verification is demonstrated.

A practitioner is trying to determine which of two interventions would be most beneficial for her client. However, she doesn't have enough time to collect baseline data. The best experimental design for her to use would be: A)a brief multiple baseline across treatments design.B)a changing criterion design.C)a multiple-probe design.D)an alternating treatments design.

D. In an alternating treatments design, collecting baseline data is not necessary in all cases. Multiple probe designs, changing criteria designs, and multiple baseline designs need baseline data to demonstrate a functional relationship between the intervention and behavior.

Elijah is excited because he is expecting to receive a card from his girlfriend in the mail. The card is expected to take 3 days to arrive. The mail comes every day at 3pm (except for Sundays). What is Elijah's rate of checking his mail? A)1 time per day.B)6 times per week.C)3 times per day.D)(Cannot determine)

D. In this question, it never says how often Elijah checked the mail. The rate of him checking his mail is the behavior of concern in this question. The fact that the mail comes at 3pm, 6 days per week is irrelevant since this is independent of his behavior. The example does not say how often he checked the mail. Therefore, the correct answer is "cannot determine".

Written consent from a guardian should be obtained: A)before a more restrictive intervention is used.B)before a new intervention is used.C)before changing an intervention goal.D)(All of the above)

D. It is necessary to gain written consent, prior to any of these scenarios.

Ralph drag races his car for a living. After pulling his car up to the starting line, he must wait for the red light to turn green before he can start accelerating or he will be disqualified. However, Ralph consistently presses the gas pedal before the light turns green, resulting in disqualification. In regards to pressing the gas pedal to accelerate, Ralph needs to work on: A)increasing his latency.B)decreasing his latency.C)decreasing his interresponse time.D)(None of the above)

D. Latency is the amount of time between the signal to begin the response (the light turning green) and the response (pressing the gas pedal). Since Ralph is consistently pressing the gas pedal BEFORE the signal to do so, latency has nothing to do with this scenario (because latency can't be measured until the light turns green). Decreasing interresponse time would mean that Ralph would press the gas pedal more frequently which doesn't make sense in this scenario. Therefore, the correct answer is "none of the above".

Landon is having trouble with his handwriting. He holds his pencil correctly, but when he writes on the paper, he does it so lightly that any markings can barely be seen. Although the letters he writes are barely visible, they appear to be written very neatly. With regards to Landon's handwriting, he should work on his: A)Duration.B)Latency.C)Topography.D)Magnitude.

D. Magnitude refers to the force or intensity of a behavior. Landon should use more force with regards to touching his pencil to the paper so that the writing shows up more clearly. "Duration" is incorrect because this refers to the total amount of time spent engaging in a behavior. "Latency" is incorrect because it refers to the time from the opportunity to respond to the response actually occurring. Finally, "topography" is less correct because it refers to the shape or form of a behavior. Since the handwriting that is visible appears to be written very nicely, focusing on pushing down harder with the pencil and making the writing more visible should be the primary concern.

The effectiveness of a reinforcer can be influenced by: A)how quickly and consistently it is delivered.B)the quality of the item.C)the quantity of the item.D)(All of the above)

D. Many factors can influence a reinforcer's effectiveness. Deprivation can make the reinforcer more potent. How quickly and consistently the reinforcer is delivered also influences a reinforcer's effectiveness. The quality of the item influences how reinforcing an item is. I would be much more likely to work for a brand new car than I would for a rusty, broken down car. The quantity (or time spent) with a reinforcer can influence its effectiveness as well.

This approach to psychology restricts the science of psychology to only those independent and dependent variables that two independent people can directly observe: A)Radical Behaviorism B)Experimental Analysis of BehaviorC)Applied Behavior AnalysisD)Methodological Behaviorism

D. Methodological behaviorism is the approach to psychology restricts the science of psychology to only those independent and dependent variables that two independent people can directly observe. Radical behaviorism is incorrect because it includes thoughts and feelings (methodological behaviorism does not). Applied behavior analysis is the science in which tactics derived from the principles of behavior are systematically to improve socially significant behavior and experimentation is used to identify the variables responsible for behavior change. Experimental analysis of behavior focuses on analyzing the basic principles that explain behavior.

Money can be a: A)generalized conditioned reinforcer.B)secondary reinforcer.C)natural reinforcer.D)(All of the above)

D. Money can be a generalized conditioned reinforcer because it can be paired with many different forms of backup reinforcers (you can use money to buy many things). Money is a secondary reinforcer because it is a learned reinforcer (it has no value to you at birth). Money can also be considered a natural reinforcer because money is used outside of the training setting in almost all places. Consider this example: you teach your client to fold towels in his home and you use contrived reinforcement in the form of candy to help establish his towel-folding behavior. Then, he gains employment at a hotel where he folds towels. Since we are no longer contriving contingencies of reinforcement via candy, money can be considered natural reinforcement. People in the natural environment earn money for doing their job. So, him working for money (just like everyone else does) could be considered natural reinforcement, since it reinforces the behavior outside of the initial training setting. Since money is a typical consequence for specific behaviors at work, it can sometimes be considered as natural reinforcement (if it is delivered quickly enough, such as being paid per part). However, money can also be a contrived reinforcer. (i.e., A parent makes a deal with their child that if he can stay quiet for 5 minutes, he will receive a dollar. Since most people do not receive money for staying quiet for 5 minutes, it would be considered a contrived reinforcer.)

A behavior analyst receives a call from a parent who is looking for a behavior analyst to work with his child. Of these answer choices, what is the FIRST thing the behavior analyst should do in this scenario? A)Conduct a behavioral interview to gain more information regarding the function of the behavior of concern.B)Build rapport with the client to establish yourself as a reinforcer.C)Observe the behavioral concern in the client's natural environment.D)Discuss the details and terms of the services you provide.

D. Of the answer choices, this would be the most appropriate first course of action. You should, as a behavior analyst, explain to the parent who you are, what kinds of services you provide, the cost of your services, how they can make a complaint to the BACB® regarding the services you provide, and you should gain the parents' consent to begin services, etc. All of these terms/details should be discussed prior to conducting any assessments or interventions.

Which of the following is/are properties of an operational definition? A)ObjectiveB)UnambiguousC)CompletenessD)(All of the above)

D. Operational definitions should include all of these properties. They should be unambiguous and objective, leaving little "grey area" on which behavior(s) fall under the definition. They should also include the boundaries of what can be included in the target behavior (and include non-examples).

When teaching a new skill, what type of reinforcement schedule is recommended? A)ContinuousB)An FR1C)Reinforcing every occurrence of the target behaviorD)(All of the above)

D. When teaching a new skill, a behavior analyst should reinforce every occurrence of the target behavior initially. All of the answer choices are synonymous.

Jerry is a behavior analyst who is a veteran in the field of ABA. He has been using a procedure to teach his clients how to wash their hands for the past 20 years. Mallory is a recently certified behavior analyst who discovered a more beneficial way to teach clients how to wash their hands. Even though Jerry has been using the same method to teach hand washing for the past 20 years, he starts using Mallory's strategy instead because the data show it is more effective. What attitude of science does this describe? A)DeterminismB)ParsimonyC)EmpiricismD)Philosophic doubt

D. Philosophic doubt means that scientists are willing to set aside their beliefs and strategies when an experiment scientifically proves that a better strategy is available. "Determinism" is incorrect since this refers to the belief that behavior is lawful and will remain unchanged if nothing in the environment changes. "Parsimony" is incorrect since this means that scientists explain phenomena with the easiest explanation available. "Empiricism" is incorrect since this refers to the idea that scientists should remain unbiased and objective during experiments.

This approach to psychology considers private events as behavior: A)Mentalism.B)Pragmatism. C)Methodological Behaviorism.D)Radical Behaviorism.

D. Radical Behaviorism is an approach to psychology that addresses everything in terms of the principles of behavior (including private events). This differs from methodological behaviorists that do not consider private events or feelings to be behavior.

Daisy walks out of class after she is asked to do schoolwork that she does not enjoy. In math class, she eloped 5 times. In science class, she eloped 2 times. In art class, she did not elope. Each class is one hour long. What is the frequency of Daisy's elopement today? A)2.33 instancesB)2.33 instances per hourC)3.5 instances per classD)7 instances

D. She eloped 5 times in math class and 2 times in science class. 5+2=7 instances. Frequency is simply the amount of times a behavior occurs. Rate is frequency divided over a unit of time. Therefore, "3.5 instances per class" and "2.33 instances per hour" are incorrect. "2.33 instances" instances is also incorrect since the behavior occurred 7 times.

Nichole runs 3 miles per day. The first day, it took her 30 minutes to run 3 miles. The second day it took her 22 minutes and on the third day it took her 23 minutes to run 3 miles. What is Nichole's average duration of running? A)75 minutesB)3 milesC)9 milesD)25 minutes

D. Since we are looking for duration in this example, we are looking for the average amount of time she spent jogging each day. She spent 30 mins, 22 mins, and 23 mins jogging, respectively. 30+22+23=75 total mins. 75/3=25 mins per run.

Reinforcer assessments include: A)concurrent schedule reinforcer assessments.B)multiple schedule reinforcer assessments.C)progressive-ratio schedule reinforcer assessments.D)(All of the above)

D. The 3 different types of reinforcer assessments are concurrent schedule reinforcer assessments, multiple schedule reinforcer assessments, and progressive-ratio schedule reinforcer assessments.

Anita is a BCBA® who recently accepted a job in Florida. Although she has lived in Minnesota her entire life, she is excited to move across the country. How long does she have to notify the BACB® of her move? A) Anita must notify the BACB® prior to her move and request that her credential be transferred to Florida. B) Anita must notify the BACB® at least 14 days prior to her anticipated move date because it typically takes several business days for the BACB® to process this address change request. C) Anita does not need to notify the BACB® of her move since the BCBA® credential is recognized nationally. D) Anita has 30 days from the date of her move to notify the BACB®.

D. The BACB® requires that you notify them within 30 days of: changing your name, address, or email address, any physical or mental conditions that impair your ability to practice behavior analysis, any fines or tickets that are related to public health or safety, or violations of the BACB's® code of ethics.

Jennifer, a female BCBA®, is getting married in 5 days and will change her last name immediately after the wedding. How long does she have to notify the BACB® of her name change? A) Jennifer has to notify the BACB® prior to her name change to avoid risk of losing her credential. B) Jennifer has 90 days from the date of her name change to notify the BACB®. C) Jennifer does not need to notify the BACB® of the name change as these records are already automatically sent to the BACB®. D) Jennifer has 30 days from the date of her name change to notify the BACB®.

D. The BACB® requires that you notify them within 30 days of: changing your name, address, or email address, any physical or mental conditions that impair your ability to practice behavior analysis, any fines or tickets that are related to public health or safety, or violations of the BACB's® code of ethics. (She does not HAVE TO tell the BACB® prior to her name change, she just has to within 30 days to avoid a violation.)

A behavior analyst is using an extinction procedure for her client's verbal aggression. What should the behavior analyst monitor for? A)Behavioral contrast in other settingsB)An extinction burstC)New undesirable behaviorsD)(All of the above)

D. The behavior analyst must watch for an extinction burst, which is a common occurrence when using extinction procedures. The behavior analyst should also monitor for behavioral contrast effects. If a behavior is put on extinction in one setting, it may occur more frequently in other settings. The behavior analyst should also watch for the possibility of new undesirable behaviors occurring. Sometimes, if an undesired behavior is suppressed, another inappropriate behavior that is part of the same functional response class may "pop up".

During a functional analysis, the child only displays the target behavior in front of the therapist. Therefore we can determine the function of the behavior is: A)access to attention.B)access to tangibles.C)escape.D)(cannot determine)

D. The child only displays the behavior in front of the therapist, so we can likely only rule out automatic reinforcement because the target behavior would likely occur in the absence of the therapist as well (during the alone condition). However, access to tangible items, access to attention, and escape would all likely involve a therapist in the room. Since any of these conditions may control the behavior, the correct answer is "cannot determine" since the information provided was inadequate for identifying the exact function of the behavior.

Behavior analysts should record behavioral data ______________ to help ensure the data is accurate. A)at the end of each dayB)at the end of each weekC)at the end of each sessionD)contemporaneously

D. The most accurate way of collecting data is to collect it as the behavior occurs. Therefore, the behavior analyst does not have to rely on their memory of past events (which is less reliable).

Sebastian enjoys throwing rocks at cars. He likes the sound the rock makes when it smashes the window out of a car. Sebastian throws rocks at cars that pass by his house, cars that pass by his school, and parked cars at a nearby grocery store.A behavior analyst wants to work with Sebastian on his rock-throwing behavior. Which experimental design should the behavior analyst use? A)A multiple baseline design across behaviorsB)A multiple baseline design across subjectsC)A multiple baseline design across settingsD)(None of the above)

D. This behavior is very dangerous. It could result in people being seriously injured. Sebastian could end up in jail, he could be sued for damages, etc. Continuing a baseline phase in this situation is unethical. Since an intervention would need to be implemented immediately, a multiple baseline design would not be ideal in this situation. (Note: If this was NOT a dangerous behavior, the behavior analyst could use a multiple baseline design across settings.)

A four-year-old is terrified of dinosaurs. Whenever she sees a dinosaur on TV, she runs and hides under her bed. This occurs about once per month. Her parents find your contact information in the phone book and ask you to help her with her fear of dinosaurs. How should you respond to this request? A) "Behavior analysts don't work with children to get them to stop being afraid of things, such as dinosaurs. Besides, it's pretty normal for a 4-year-old to be scared of these kinds of things." Incorrect answer B) "I'll print off some data sheets and bring them to your home. Let's begin tracking this behavior. If she runs and hides under her bed, rub her back and tell her dinosaurs are extinct. I'll come back in a week to analyze this data and then we'll decide if I'll be a good fit to work with her." C) "Make her a points board where at the end of each day, she receives a star if she does not act scared of dinosaurs. Then she can trade these stars in for items she enjoys. It should get better over time." Missed correct answer D) (None of the above)

D. This is a tricky one, but behavior analysts cannot provide any advice on behavioral interventions until they have discussed the terms of their relationship, discussed the cost of services, etc. Therefore the answers that offer behavioral advice are incorrect. Behavior analysts do use systematic desensitization to make things less scary for children, so the top answer is also incorrect.

Which of the following is NOT a distinguishing feature of single-subject research designs? A)Individuals serve as their own control.B)They allow for repeated measures.C)They allow for prediction, verification, and replication.D)They can only be conducted with one subject.

D. This is not a distinguishing characteristic of single-subject research designs. Single-subject research designs can be conducted with one subject, or a small group of subjects. The reason they are called single-subject designs is because each individual's data is tracked, rather than an average of the entire group's data. Therefore, individuals serve as their own control. This allows for repeated measures of the behavior throughout the research study. It also allows us to predict, verify, and replicate the experimental results.

All of the following are appropriate reasons to terminate services with a client, EXCEPT: A)the client is benefiting from your services and has met his treatment goals.B)the client no longer wants to receive services.C)the client is not benefiting from your services.D)the client has met his treatment goals and new treatment goals have been established.

D. This is not an appropriate reason to terminate services. Oftentimes, treatment goals change over time. A treatment goal (such as paying for items at a store) can progress to more complex skills (such as paying for a bus ticket, navigating the community, going into a store independently and purchasing items on a grocery list). Just because initial goals have been met, it does not necessarily mean that your services are no longer needed. The behavior analyst should determine whether or not additional treatment goals will be created (after the original goals are met) prior to terminating services. If the client or his guardian requests that services be discontinued, a behavior analyst should grant this request. If all treatment goals are met and no new goals are established, the behavior analyst can discontinue services. If the client is not benefiting from your services (even after taking multiple approaches, reasonable accommodations, etc.), discontinuing services may be appropriate.

Gracie overhears her parents talking about her teacher. Her mom says, "I hate Gracie's teacher! She is letting her fall behind in class. I hope she gets fired!" The next day Gracie goes to school and tells her teacher, "My mom hates you and hopes you get fired!" Gracie's verbal behavior is an example of: A)an echoic.B)a mand.C)an intraverbal.D)(None of the above)

D. This is not an echoic because Gracie did not say the exact same thing as her mother; she worded it differently. This is not a mand because Gracie is not requesting/asking for anything. Finally, this is not an intraverbal since Gracie initiated the conversation (an intraverbal would be evoked by the teacher speaking first). Her verbal behavior is best described as a tact. Therefore, the correct answer is "none of the above".

Which explanation of behavior would most likely be made by a radical behavior analyst? A)Jimmy gets frustrated when the other children don't play with him.B)Jimmy runs around the classroom because he eats too much sugar at breakfast time. C)Jimmy enjoys reading over math because he engages in fewer problem behaviors during reading time. D)Jimmy's environment does not provide enough reinforcement and he engages in the problem behavior to receive attention.

D. This is the most correct choice for the explanation of behavior from the perspective of a radical behavior analyst.

Which of the following is a threat to internal validity? A)Confounding variablesB)MaturationC)Low treatment integrityD)(All of the above)

D. Threats to internal validity can include: confounding variables, maturation, setting confounds, measurement confounds, poor operational definitions, and low treatment integrity.

Violet, a BCBA®, is having a hard time with a client. She has asked colleagues but requires more support. She has decided to post her question on a Facebook group. She carefully didn't put any personal indicators and was delighted to get a lot of feedback from many behavior analysts in the group. What should Violet be mindful of? A) Those giving advice may not be credentialed to provide recommendations. B) It is inappropriate to provide recommendations without knowing the situation (i.e., the client, the program, and all the nuances of the environment). C) Without data, what could anyone possibly be advising Violet to do. D) All of the above

D. When behavior analysts provide public statements, advice, or comments by means of public lectures, demonstrations, radio or television programs, electronic media, articles, mailed material, or other media, they take reasonable precautions to ensure that (1) the statements are based on appropriate behavior-analytic literature and practice, (2) the statements are otherwise consistent with this Code, and (3) the advice or comment does not create an agreement for service with the recipient.

Diego is tracking how many miles he has ran this year, using a cumulative record. At the end of the year, how will Diego determine how many total miles he ran? A)By adding up all of the data points on the graphB)By adding up all of the data points on the graph, and then dividing that number by the number of data pointsC)By adding up all of the data points on the graph, then dividing that number by the number of data points, and then multiplying that number by 100D)By looking at the last data point of the cumulative record

D. When using a cumulative graph, Diego would keep track of the total number of miles he ran by adding how many miles he jogged that day on top of the total amount of miles he ran earlier in the year. To find the final/total occurrence of the behavior on a cumulative record, you simply have to look at the last data point to determine the grand total. Therefore, if Diego ran 230 miles this year, the last data point would be at 230 miles.

You are using a momentary time sampling data collection system to monitor your client's verbal aggression. Each interval is 15 minutes long. 5 minutes after you recorded data for the 9th interval, you receive a phone call that lasts 5 minutes. You are off the phone with 5 minutes left in the 10th interval.What should you do with the data you recorded? A) Discard the entire data sheet for that day. B) Put an asterisk next to the 10th interval to remind yourself to void out this interval when calculating the data. C) Redo the 10th interval. Correct answer D) (None of the above)

D. When using momentary time sampling, you simply observe the subject briefly at THE END OF THE INTERVAL. Then you mark whether or not the behavior was occurring at the exact moment in time. In this example, you had 15 minutes of "spare time" between the 9th interval and the end of the 10th interval. Because you were able to observe the end of the 10th interval (even though you took a phone call in the middle of the interval), you did not miss collecting data for any of the intervals. Therefore, it is unnecessary to void out the interval or void out the entire data sheet. It is also unnecessary to redo the entire interval. Not having to observe the behavior for the entire interval is one of the benefits of using momentary time sampling.

When you begin working with a client, it is necessary to tell them: A) how to make a complaint to the BACB® regarding the services you provide. B) how to discontinue services if desired. C) how much services cost and what methods of payment you accept. D) (All of the above)

D. When you first meet the client, you should provide them with contact information, discuss cost of services, how to discontinue services, how to file a complaint, etc.

You are a behavior analyst visiting your client at his school. You are collecting partial-interval data on his hand-raising behavior. During the first few seconds of a 15-minute interval, your client raises his hand. Right after you record that the response occurred, you go to the restroom. You make it back before the next interval begins. Since you went to the restroom in the middle of an interval, what should you do with the data you collected this session? A)Void the entire datasheet for this session since you were absent during a portion of one interval.B)Don't dispose of the datasheet for this session. Simply void out the interval in which you went to the restroom since you were not absent for any of the other intervals.C)Don't dispose of the datasheet since you were only absent during one interval and the likelihood of this negatively impacting the entire outcome of the intervention is minimal.D)Don't dispose of the datasheet since you did not need to observe the rest of that interval.

D. You are using partial-interval recording to record whether or not the target response occurs at any point during the 15-minute interval. Since the target response occurred at the beginning of the interval, you do not need to observe the rest of that interval (as long as you are back in time for the beginning of the next interval). The data is not negatively impacted by you going to the restroom in this scenario.

You are a behavior analyst who works with a client, named Rita. You have worked with Rita for nearly 2 years. Rita has recently become physically aggressive towards her parents. You have no history of working with clients who are physically aggressive. What could you do in this scenario? A) Refer Rita to someone who has the relevant experience and expertise with physical aggression and discontinue your services. B) Ask a behavior analyst, who is competent in working with people who are physically aggressive, to address this behavior. Continue to provide services for the behavioral concerns you were already addressing. C) Ask a behavior analyst, who is competent in working with people who are physically aggressive, to supervise you through this case and teach you intervention strategies that may be effective. D) (All of the above)

D. You could take any/all of these courses of action in this scenario. If you have never worked with people who are physically aggressive, it may be unethical for you to work on this behavior without the supervision of another behavior analyst. You could simply transfer the client to someone who does have the relevant training and experience. You could reach out to a behavior analyst who is competent in this area to see if they will supervise you. In some cases, it may be possible for 2 behavior analysts to work with the same client if she engages in multiple behavioral problems and each behavior analyst addresses the problem behaviors that they are competent with.

You sincerely believe that your client intends to injure his next door neighbor. When you ask your client if he is going to intentionally hurt his neighbor, he says, "No." Are you allowed to tell the neighbor that your client may injure him? A)No, this would be considered a violation of HIPAA and is considered unethical. You would have to ask your client if you can tell the neighbor he may be in danger.B)No. Even if you believe your client may injure his neighbor, he said he would not. Therefore, you cannot tell the neighbor.C)No, you cannot reveal your client's identity unless ordered to by a judge.D)Yes, you may disclose confidential information to protect others from harm.

D. You do not have to gain permission from your client to talk to the neighbor if you have reason to believe the neighbor may be injured by your client (even if the client says he will not injure the neighbor). Generally, you would need permission from the client or to be ordered by a judge to release confidential information. But when it comes to protecting others from a serious risk of danger, confidential information may be disclosed.

Under which circumstance(s) could a behavior analyst reveal the identity of their client, even if the client does not consent? A)If the client's records are subpoenaed to court.B)If your client threatens to kill someone.C)If making a referral for another professional service for your client.D)(All of the above)

D. here are only a few instances where a behavior analyst could reveal confidential information about their client without the client's (or their guardian's) consent. You may reveal your client's identity if you believe the client may injure or kill someone. You must reveal your client's identity if you are subpoenaed to court. Finally, if you are seeking other professional services for your client, it may be possible for you to disclose their identity.

Mr. Smith catches his teenage son smoking cigarettes. Mr. Smith takes the cigarettes and rips them up. His son cries and says he will never smoke again. This is an example of: A)negative punishment. B)positive punishment. C)negative reinforcement. D)Cannot determine

D. imply because his son says he will not smoke again, does not mean that he truly will quit. Since the scenario did not say that his son stopped smoking in the future, we cannot determine that Mr. Smith's actions were in fact a punisher.

Clarissa works at a vacuum factory. Today, her boss said, "For every black vacuum you assemble, you will receive $20. For every grey vacuum you assemble, you will receive $15." It takes about 1 hour to build a vacuum, no matter the color. It also takes the same amount of effort to build a black and grey vacuum. Clarissa builds 4 black vacuums today. According to the Matching Law, Clarissa most likely built _______________ grey vacuums today. A)0B)2C)3D)5

The matching law refers to the distribution of responses when there is more than one schedule of reinforcement available at the same time. With concurrent variable interval or concurrent fixed interval schedules of reinforcement, the person is likely to distribute their responding proportionally between the two schedules of reinforcement. With concurrent variable ratio or concurrent fixed ratio schedules of reinforcement, the person is likely to allocate all of their responses to the highest level of reinforcement. Since Clarissa's reinforcement schedule is on a fixed ratio schedule (one black vacuum=$20 and one grey vacuum=$15), she is likely to allocate all of her time to build black vacuums. Therefore, according to the Matching Law, Clarissa likely built 0 grey vacuums this day.


Set pelajaran terkait

Precalc Polar Coordinates (Unit 14)

View Set

اساسيات التخطيط - الجزء الاول: التخط

View Set

Chapter 36: Introduction to the Nervous System

View Set

US History Ch. 2 Study Guide: Transplantations and Borderlands

View Set